Вы находитесь на странице: 1из 86

IIT-JEE | AIPM T | AIEEE | OLYM PIADS | KVPY | NTSE

“KVPY PAPERS & SOLUTIONS

OF LAST 4 YEARS”

TO DOWN LOAD VISIT :


http://dlpd.resonance.ac.in
GENERAL INSTRUCTIONS FOR KVPY TEST PAPER
YEAR : 2007

Time : 3Hr. Max. Marks : 100

1. In Year 2007, Question Number 1-50 each carries 1 (one) mark, Maths(1-20), Physics

(21-30), Chemistry(31-40), Biology (41-50) & Question Number 51 to 75 each carries

2 marks, Maths(51-60), Physics( 61-65), Chemistry (66-70), Biology(71-75)

2. Blank papers, clip boards, log tables, slide rule, calculators, mobile or any other
electronic gadgets in any form is not allowed.

3. Write your Name and Roll No. in the space provided in the bottom of this booklet.

4. Before answering the paper, fill up the required details in the blank space provided in

the answer sheet.

5. Do not forget to mention your roll number neatly and clearly in the blank space provided
in the answer sheet.

6. In case of any dispute, the answer filled in the OMR sheet available with the institute

shall be final.
YEAR-2007 (KVPY-STREAM-SA)

PART-I (1 Mark)
MATHEMATICS

1. Let s1 (n) be the sum of the first n terms of the arithmetic progression 8, 12, 16, ....... and let s2(n) be the
sum of the first n terms of arithmetic progression 17, 19, 21 ......... If for some value of n, s1(n) = s2(n) then
this common sum is :
(A) not uniquely determinable (B) 260
(C) 216 (D) 200

2. The number of solutions are in [0, 2] such that (sin(2x))4 = 1/8 is
(A) 2 (B) 4 (C) 8 (D) 16

3. The set of real numbers r satisfying :

3.r 2 – 8.r  5
0
4.r 2 – 3.r  7
is
(A) the set of all real numbers
(B) the set of all positive real numbers
(C) the set of all real numbers strictly between 1 and 5/3
(D) the set of all real numbers which are either less than 1 or greater than 5/3

4. If a, b and c are distinct real numbers such that a : (b + c) = b : (c + a) then


(A) a, b, c are all positive (B) a, b, c are all negative
(C) a + b + c = 0 (D) ab + bc + ca + 1 = 0
x
5. How many positive real numbers x are there such that x x x
 ?
 x x

(A) 1 (B) 2 (C) 4 (D) infinite

6. Let [x] denote the greatest integer part of a real number x. If

40
 n2 
M   
 2 
n 1 

then M equals
(A) 5700 (B) 5720 (C) 5740 (D) 11060

7. Suppose for fixed real numbers a and b.f(x) = x3 + ax2 + bx + c


has 3 distinct roots for c = 0. Then
(A) fc(x) has 3 distinct roots for all real c
(B) fc(x) has 3 distinct roots for all real c > 0 or for all real c < 0, but not for both
(C) fc(x) has 3 distinct roots for all real c in (p, q) for some p < 0 and q > 0
(D) fc(x) need not have 3 distinct roots for any real c other than zero

PAGE # 1 1
8. Given a semicircle of radius 1, let a be the side of an equilateral triangle which is inscribed in the
semicircle with its vertices on the boundary of the semicircle (boundary includes the bounding diameter
also). Then the set of possible values of a is
(A) {1}

2
(B) {1, }
3

2
(C) the set of all positive real numbers not exceeding
3
2
(D) the set of all real numbers which are greater than or equal to 1, but less than or equal to
3

9. A ray of light originating at the vertex A of a square ABCD passes through the vertex B after getting
reflected by BC, CD and DA in that order. If  is the angle of the initial position of the ray with AB then sin
equals.

(A) 2 / 13 (B) 3 / 13 (C) 3/5 (D) 4/5

10. Let ABC be triangle with AB = AC = 6. If the circumradius of the triangle is 5, then BC equals
(A) 25/3 (B) 9 (C) 48/5 (D) 10

11. Let D and E be points on sides AB and AC of a triangle ABC such that
(A) DE is parallel to BC
(B) DE divides the area of the triangle ABC into two equal parts.
The ratio of the distance from A to DE to the distance between DE and BC is

(A) 1 : 1 (B) 2 : 1 (C) 2 :1 (D) 2 +1:1

12. If  and  are acute angles such that cos2 + cos2 = 3/2 and sin . sin  = 1/4, then  +  equals
(A) 30° (B) 45° (C) 60° (D) 90°

13. There is a point inside an equilateral triangle which is at distances 1, 2 and 3 from the three sides. The
area of the triangle is
(A) not uniquely determinable (B) 6

(C) 6 3 (D) 12 3

14. The wealth of a person A equals the sum of that of B and C . If he distributes half of his wealth between
B and C in the ratio 2 : 1 then the wealth of B equals the sum of that of A and C. Then the fraction of the
wealth that A should distribute between B and C in the ratio 1 : 2 so that the wealth of C equals the sum
of that of A and B is
(A) 1/2 (B) 2/3 (C) 3/4 (D) 1

15. You have a measuring cup with capacity 25 ml and another with capacity 110 ml, the cups have no
markings showing intermediate volumes. Using large container and as much tap water as you wish.
What is the smallest amount of water you can measure accurately ?
(A) 1 ml (B) 5 ml (C) 10 ml (D) 25 ml

PAGE # 2 2
16. A person travels in a car from his village to a town at a speed of 50 kms/hr for four hours. There he spends
an hour moving about 20 kms. He then returns to his village at a speed 40 kms/hr. His average speed over
the whole journey (in kms/hr) is

4 2
(A) 44 (B) 42 (C) 44 (D) 46
9 3
17. There are 20 unit cubes all of whose faces are white, and 44 unit cubes all of whose faces are red. They
are put together to form a bigger cube (4 × 4 × 4). What is the minimum number of white faces visible on
this larger cube ?
(A) 20 (B) 14 (C) 12 (D) 8

18. The number of 2-digit numbers n such that 3 divides n – 2 and 5 divides n – 3 is
(A) 5 (B) 6 (C) 7 (D) 10

19. Let m be the number of ways in which two couples can be seated on 4 chairs in a row so that no wife is
next to her husband and n be the number of ways in which they can be seated in 4 chairs in a circle. In
the latter case, rotations are considered different configurations. Then
(A) m = n (B) m = 2n (C) m = 4n (D) m = 8n

20. A certain school has 300 students. Every student reads 5 newspapers and every newspaper is read by 60
students. Then the number of newspapers.
(A) is at least 30 (B) is at most 20
(C) is exactly 25 (D) cannot be determined by the data

PHYSICS

21. A fully loaded elevator has a mass of 6000 kg. The tension in the cable as the elevator is accelerated
downward with an acceleration of 2ms–2 is (Take g = I0 ms–2)
(A) 7·2 × 104 N (B) 4.8 × 104 N (C) 6 × 104 N (D) 1.2 × 104 N

22. A body is moving with constant acceleration from A to B in a straight line. C is the mid-point of AB. If u and
v are the speeds at A and B respectively. The speed at C is

uv v–u u2  v 2 v 2 – u2
(A) (B) (C) (D)
2 2 2 2

23.
, A ray of light enters a slab of material with increasing refractive index. Four possibilities of the trajectory
of the ray are shown below.

Slab with
increasing
Refractive
index

A B C D

The correct choice is :


(A) A (B) B (C) C ( D) D

PAGE # 3 3
238 206
24. The isotope U92 decays through a chain of  and  emissions ending up at the product Pb82 . This

chain involves
(A) 8  emission and 6  emissions
(B) 8  emission and 8  emissions
(C) 6  emission and 8  emissions
(D) 6  emission and 6  emissions

25. Figure shows an assembly consisting of a number of pendulums of varying lengths. The driver pendulum is
pulled aside and released so that it oscillates in a plane perpendicular to that of the diagram. It will be
observed that

String

Driver
pendulum

(A) all pendulums oscillate with the frequency of the driver pendulum and have the same amplitude.
(B) pendulums oscillate with different frequencies but equal amplitude, the shortest pendulum oscillating
with the highest frequency.
(C) all pendulums oscillate with the frequency of the driver pendulum; the pendulum with length equal to
that of the driver has the greatest amplitude.
(D) pendulums oscillate with different frequencies and different amplitudes.

26. You are riding on your bicycle with inflated tyres. Your friend asks for a lift and sits on the carrier behind
you :
(A) The air pressure in the tyres increases.
(B) The air pressure in the tyres decreases.
(C) The air pressure in the tyres remains the same.
(D) Nothing in the system changes except the reaction of the ground

27. The circuit shown has 3 identical light bulbs A, B, C and 2 identical batteries E1, E2 . When the switch is
open, A and B glow with equal brightness. When the switch is closed:

C
E1 A

S
E2 B

(A) A and B will maintain their brightness and C will be dimmer than A and B.
(B) A and B will become dimmer and C will be brighter than A and B.
(C) A and B will maintain their brightness and C will not glow.
(D) A, B and C will be equally bright.

PAGE # 4 4
28. A boat at rest has two persons of unequal weights seated at either end. If they walk across the boat and
interchange their positions,
(A) the boat will move in the direction of walking of the lighter person.
(B) the boat will move in the direction of walking of the heavier person.
(C) the boat will not move at all.
(D) the boat will oscillate about a mean position and stop once the persons have moved to the new
positions.

29. A physical quantity that is conserved in a process


(A) must have the same value for all observers.
(B) can never take negative values.
(C) must be dimensionless.
(D) need not necessarily be a scalar.

30. A coolant in a chemical or nuclear plant is a liquid that is used to prevent different parts of a plant from
getting too hot. One important property of coolant is that it
(A) should have low specific heat.
(B) should have high specific heat.
(C) high density and low thermal conductivity.
(D) low density and high thermal conductivity.

CHEMISTRY

60 60
31. The half-life of 27 Co is 5.3 years. How much of 20 g of 27 Co will remain radioactive after 21.2 years ?

(A) 10 g (B) 1.25 g (C) 2.5 g (D) 3.0 g

32. The element which reacts both with acids and bases to produce a salt and hydrogen gas is -
(A) Zn (B) Mg (C) Li (D) S

33. The compound having a triple bond is -


(A) Benzene (B) Cyclohexane (C) Acetylene (D) Glucose

34. An element which has 2 electrons less than that in Ne is -


(A) Mg (B) Na (C) O (D) F

35. Chlorine gas is produced from HCI by the addition of -


(A) K2SO4 (B) KCl (C) KNO3 (D) KMnO4

36. Among HCl, HBr, HF, and HI the weakest acid in water is -
(A) HCl (B) HF (C) HBr (D) HI

37. Products of the reaction Ca(OH)2 + 2NH4Cl, are


(A) CaCI2 + 2NH3 + 2H2O (B) CaCl2 + NH4OH + H2
(C) CaO + 2NH3 + 2H2O (D) CaO + NH4OH + H2

PAGE # 5 5
38. When a few drops of concentrated sulphuric acid are added to a few drops of a clear sugar (sucrose)
solution in a test tube, the solution
(A) remains colorless (B) turns black (C) turns blue (D) turns white

39. The reaction ZnO + C  Zn + CO is an example of a


(A) Combination reaction (B) Reduction-oxidation reaction
(C) Displacement reaction (D) Decomposition Reaction

40. The monomer of Teflon is :


F F
(A) (B) (C) (D)
CN F F
Me Cl

BIOLOGY
41. In a wound, 'pus' is a :
(A) A mixture of destroyed germs, killed leucocytes and damaged tissue cells
(B) Concentrated blood plasma
(C) Thick mucus secretion
(D) Concentrated secretion of the sebaceous gland

42. Discovery of fossils has provided biologists with significant clues as in the steps in evolution. Archaeop-
teryx one such example, which is connection link between :
(A) Pisces and amphibian (B) Amphibia and reptilia
(C) Reptilia and aves (D) Aves and mammalian

43. Which of the following is a bacterial disease ?


(A) Smallpox (B) Measles (C) Meningitis (D) Rabies

44. During photosynthesis, light energy :


(A) Is converted to chemical energy (B) Is converted to kinetic energy
(C) Is the catalyst (D) Dissociates CO2 directly

45. Majority of the world population depends largely on the following for their staple food :
(A) Fungi and algae (B) Animal products
(C) Dicotyledenous (D) Monocotyledenous plants

46. If you put a nail at the middle of a young plant and allow it to grow, then :
(A) You will find the nail at the same position after the tree is fully grown
(B) The nail will fall out
(C) The nail will rise along with the tree
(D) The nail will come closer to the ground

47. If you are asked to write on a topic of interest, you can read as many books as you want but never copy
someone else's writing on the same topic. The act is called plagiarism and is morally incorrect, as :
(A) You are stealing someone else's labor (B) It is a punishable offence in court
(C) You have not paid money to do so (D) A patent exists on the writing

PAGE # 6 6
48. A couple has 8 children, all boys. What is the chance that their ninth child will be another boy ?
(A) 1/2 (B) 1/4 (C) 1/8 (D) 1/16

49. Grapes were put in a concentrated sugar solution. On examination after 12 hours, the grapes were
shrunken. This is because :
(A) Grapes become sweeter (B) Water evaporates from the solution
(C) Sugar induces disintegration of grapes (D) Loss of water from grapes by osmosis

50. On a normal sunny day, rate of photosynthesis (per unit time) is maximum during :
(A) Early morning (B) Between late morning to before noon
(C) Midday (D) Late evening

PART-II (2 Mark)
MATHEMATICS

51. The number of distinct pairs (x, y) of the real numbers satisfying x = x3 + y4 and y = 2xy is :
(A) 5 (B) 12 (C) 3 (D) 7

52. We want to find a polynomial f(x) of degree n such that f(1) = 2 and f(3) = . Which of the following is
true ?
(A) There does not exist such a polynomial
(B) There is exactly one such polynomial and it has degree 1
(C) There are infinitely many such polynomials for each n  1
(D) There are infinitely many such polynomials for each n  2 but not infinitely many for n = 1

53. A polynomial p(x) when divided by x2 – 3x + 2 leaves remainder 2x – 3. Then :


(A) p(x) must have a root between 0 and 3
(B) p(x) cannot have a root between 0 and 3
(C) p(x) must have a real root but may or may not be between 0 and 3
(D) p(x) need not have a real root

54. In a rectangle ABCD , M and N are the midpoints of the sides BC and CD respectively. If ANM = 90° then
AB : BC is :

(A) 2 : 1 (B) 2 :1 (C) 3 :1 (D) 1: 1

55. Consider the triangle OAB in the xy - plane where O = (0, 0), A = (6, 0), B =  2,3 . A square PQRS is
inscribed in the square with P, Q on OA, R on AB and S on BO. Then the side of the square equals:

9 3 5
(A) 3 / 2 (B) (C) (D) 2
4 2 2

56. Let a semicircle with centre O and diameter AB. Let P and Q be points on the semicircle and R be a point
on AB extended such that OA = QR < PR. If POA = 102° then PRA is :
(A) 51° (B) 34°
(C) 25.5° (D) not possible to be determined

PAGE # 7 7
57. Let A, B, C, D be collinear points in that order. Suppose AB : CD = 3 : 2 and BC : AD = 1 : 5. Then AC :
BD is :
(A) 1 : 1 (B) 11 : 10 (C) 16 : 11 (D) 17 : 13

58. Given a sphere, a cone is constructed so that the cone and the sphere have the same volume, but the
total surface area of the cone is k times that of the sphere, where k is determined so that there is a unique
cone satisfying this property. Then k equals :

(A) 1 (B) 2 (C) 3 (D) 3


2

59. How many 4-digit numbers are there with the property that it is a square and the number obtained by
increasing all its digits by 1 is also a square ?
(A) 0 (B) 1 (C) 2 (D) 4

60. The number of positive fractions m/n such that 1/3 < m/n < 1 and having the property that the fraction
remains the same by adding some positive integer to the numerator and multiplying the denominator by
the same positive integer is :
(A) 1 (B) 3 (C) 6 (D) infinite

PHYSICS

61. The rear wheels of a car are turning at an angular speed of 60 rad/s. The brakes are applied for 5s,
causing a uniform angular retardation of 8 rads–2. The number of revolutions turned by the rear wheels
during the braking period is about
(A) 48 (B) 96 (C) 32 (D) 12

62. The energy released in a modest size atomic bomb (20 kiloton) is about 1014 J. On a monsoon day in
Mumbai, there was a heavy rainfall of about 100 cm over an area of about 100 km2. The energy released
in the atmosphere on that day is roughly equivalent to that released in
(A) 2000 atomic bombs. (B) 100 atomic bombs
(C) atomic bomb (D) negligible compared to an atomic bomb.

63. A battery of 10 V and negligible internal resistance is connected across the diagonally opposite corncr of
a cubical network consisting of 12 resistors each of resistance 1 . The total current I in the circuit
external to the network is :

10V

(A) 0.83 A (B) 12 A (C) 1 A (D) 4 A

PAGE # 8 8
64. Light incident on a rotating mirror M is reflected to a fixed mirror N placed 22.5 km away from M. The fixed
mirror reflects it back to M (along the same path) which in turn reflects the light again along a direction
that makes an angle of 27° with the incident direction. The speed of rotation of the mirror is:
(A) 250 revolutions s–1 (B) 500 revolutions s–1
(C) 1000 revolutions s–1 (D) 125 revolutions s–1

65. In aerial mapping a camera uses a lens with a 100 cm focal length. The height at which the airplane must
fly, so that the photograph of a 1 km long strip on the graund fits exactly on Ihe 20 cm long filmstrip of the
camera, is:
(A) 200 km (B) 20km (C) 5 km (D) 1 km

CHEMISTRY

66. A 3 N solution of H2SO4 in water is prepared from Conc. H2SO4 (36 N) by diluting -
(A) 20 ml of the conc. H2SO4 to 240 ml (B) 10 ml of the conc. H2SO4 to 240 ml
(C) 1 ml of the conc. H2SO4 to 36 ml (D) 20 ml of the conc. H2SO4 to 36 ml

67. The gas responsible for under water corrosion of iron and the product of corrosion, respectively, are -
(A) Dissolved oxygen and Fe2O3 (B) Dissolved hydrogen and Fe2O3
(C) Dissolved oxygen and Fe(OH)3 (D) Dissolved nitrogen and Fe(OH)3

68. The solubility curve of KNO3 as a function of temperature is given below

250
Solubility (g/100 ml water)

200

150

100

50

0 20 40 60 80 100
Temperature (°C)

The amount of KNO3 that will crystallize when a saturated solution of KNO3 in 100 ml of water is cooled
from 90°C to 30 °C, is
(A) 16 g (B) 100 g (C) 56 g (D)160 g

69. In the reaction.


yeast X KMnO 4 Y
Glucose     
the products X and Y, respectively, are -

(A) EtOH, CH3COOH (B) MeOH, HCOOH


(C) EtOH, CH3CHO (D) EtOH, HOCH2CH2OH

PAGE # 9 9
70. A sample of water was checked for suitability for drinking and was subjected to a chemical test. Pure zinc
granules and sulphuric acid were added to the water sample. The effervescence that resulted from the
reaction was bubbled through a tube containing lead acetate solution. A black precipitate appeared. The
outgoing gas was subsequently passed through a heated copper tube, a black mirror appeared on the
wall of the tube. The first and second black substances, respectively, are :
(A) HgS and CuS (B) PbS and CuS (C) As and Hg (D) PbS and As

BIOLOGY

71. A swimmer crossing the British channel after 2 hrs of vigorous swimming experiences severe muscle
armors and is forced to discontinue. Which of the following options given' below could give rise to this
problem :
(A) Muscle tear (B) Sea winter diffusion into muscles
(C) Bite of pirranahas (D) Lactic acid accumulation

72. In an experiment to monitor the ascent of sap, the following treatments were carried out to a twig dipped
in water. In which case wilting will be observed ?
(A) Bark removed (B) Pith removed
(C) Twig cut into half (D) Glucose added to water

73. Root pressure on a tree is typically about 2-6 atm. This is sufficient to raise the water level upto a few feet.
Tall trees get water at the top due to :
(A) Capillary rise and suction (B) A pump operating in the growing tree
(C) Fed by rain water (D) Water content in the atmosphere

74. A patient given broad spectrum antibiotic suffered from vitamin deficiency because :
(A) Antibiotic inactivated the vitamins
(B) Antibiotic inhibited the synthesis of vitamins
(C) Antibiotics killed the commensals in the gut which produced vitamins
(D) Antibiotics killed the vitamin producing cells in liver

75. Partial removal of liver is not harmful because :


(A) Liver being a large organ can suffice the functions even if a part is removed
(B) Liver is not a very essential organ of the body
(C) Liver has regenerative capacity and will grow after partial hepatectomy
(D) The function of liver can be taken over by kidneys

PAGE # 1010
YEAR-2008 (KVPY-STREAM-SA)

PART-I (1 Mark)
MATHEMATICS

1. If a and b are any two real numbers with opposite signs, which of the following is the greatest ?
(A) (a–b)2 (B) (|a| – |b|)2 (C) |a2 – b2| (D) a2 + b2

2. The sum of the infinite series :

1 2 3 n
   ...  n  ....
10 10 2 10 3 10

1 10 1 17
(A) (B) (C) (D)
9 81 8 22

3. The number (1024)1024 is obtained by raising (16)16 to the power n. What is the value of n ?
(A) 64 (B) 642 (C) 6464 (D) 160

4. The smallest value the expression x2 + 6x+ 8 attains on the set { x  R | x2 – 2x – 8  0} is


(A) 0 (B) –1 (C) 8 (D) 3

5. Let P1 be the set of all prime numbers, i.e., P1 = {2, 3, 5, 7,11, . . .}, Let Pn = {np | p  P1|}, i.e., the set
of all prime multiples of n. Then which of the following sets is non empty ?

(A) P1  P23 (B) P7  P21 (C) P12  P20 (D) P20  P24

6. The number of integers a such that 1  a  100 and aa is a perfect square is :


(A) 50 (B) 53 (C) 55 (D) 56

7. On a card, the following three statements are found :


(1) On this card exactly one statement is false.
(2) On this card exactly two statements arc false.
(3) On this card exactly three statements are false.

The number of false statements on the card is exactly


(A) 0 (B) 1 (C) 2 (D) 3

8. In triangle ABC, with A = 90°, the bisectors of the angles B and C meet at P. The distance from P to the

hypotenuse is 4 2 . The distance AP is :

(A) 8 (B) 4 (C) 8 2 (D) 4 2

PAGE # 1111
9. In a rhombus one of the diagonals is twice the other diagonal. Let A be the area of the rhombus in square
units. Then each side of the rhombus is :

1 1 1
(A) A (B) 2A (C) 5A (D) 4A
2 2 4

10. In the following figure, AE = EB, BD = 2DC What is the ratio of the areas of PED and ABC ?

E
P

B C
D

1 1 1 1
(A) (B) (C) (D)
4 6 9 12

11. Let P be a point in the interior of the rectangle ABCD Which of the following sets of numbers can form the
areas of the four triangles PAB, PBC, PCD, PDA in same order ?
(A) 10,9,12,5 (B) 21,15,6,12 (C) 10,9,8,6 (D) 12,8,7,5

12. The sum of 7 consecutive positive integers is equal to the sum of the next five consecutive integers. What
is the largest among the 12 numbers ?
(A) 24 (B) 23 (C) 22 (D 21

13. A large tanker can be filled by two pipes A and B in 60 minutes and 40 minutes respectively. How many
minutes will it take to fill the empty tanker if only B is used in the first-half of the time and A and B are both
used in the second-half of the time ?
(A) 15 (B) 20 (C) 27.5 (D) 30

14. Let 0 < a < b < c be 3 distinct digits. The sum of all 3-digit numbers formed by using all the 3 digits once
each is 1554. The value of c is :
(A) 3 (B) 4 (C) 5 (D) 6

15. Let a sequence have 200 zeroes to start with. In step 1, to every position in the sequence we add1. In
step2, to every even position in the sequence we add1. In step 3, to every position which is a multiple of
3, we add 1. This is continued up to 200-th step. After the 200-th step, what will be the value in the 128th
position ?
(A) 8 (B ) 16 (C) 128 (D) 200

PAGE # 1212
PHYSICS
16. A ball is thrown vertically upwards with a certain initial velocity. Assume that there is no resistance due
to air. Among the graphs below, the graph that is not an appropriate representation of the motion of the
ball is :

Potential Energy
Potential Energy
Kinetic Energy

Kinetic Energy
(i) (ii) ) (iii) (iv)

Time Time Height of the ball Speed of the ball

(A) A (B) B (C) C (D) D

17. An electron of mass me initially at rest takes time t1 to move a distance s in a uniform electric field in the
same field environment, a proton of mass mp initially at rest takes time t2 to move the same distance (in
the opposite direction). lgnoring gravity, the ratio t2/t1 is :

1/ 2 1/ 2
 me  mp  mp 
(A) 1 (B)   (C) (D)  

 mp 
 me  me 

18. A simple camera with a converging lens of 60 mm focal length is focused on very far objects. To focus the
camera on a nearby object 1.5 m away, the distance between the film and lens will have to be:
(A) decreased by 2.5 mm
(B) increased by 2.5 mm
(C) kept fixed as before. but aperture increased by a factor of 2.5
(D) kept fixed as before, hut aperture decreased by a factor of 2.5

19. A molecule of gas in a container hits one wall (1) normally and rebounds back. It suffers no collision and
hits the opposite wall (2) which is at an angle of 30º with wall 1.

Wall 1

30º
Wall 2

Assuming the collisions to be elastic and the small collision time to be the same for both the walls, the
magnitude of average force by wall 2. (F2) provided to the molecule during collision satisfy :

(A) F1 > F2 (B) F1 < F2


(C) F1 = F2, both non-zero (D) F1 = F2 = 0

PAGE # 1313
20. A stone dropped from the window of a stationary train hits the ground and comes to rest. An identical
stone is dropped from the window when the same train is moving with speed v and it comes to rest on the
ground. Assume that in each case, the entire energy lost in impact goes into heating the stone. Then

(A) The first stone is is slightly more heated than the second.
(B) The second stone is slightly more heated than the first.
(C) Both the stones will be raised to the same slightly higher) temperature.
(D) The second stone will be slightly more heated than the first only if its horizontal speed during fall is
more than the final vertical speed.

21. A negatively charged particle initially at rest is placed in an electric field that varies from point to point.
There are no other fields. Then :
(A) the particle moves along the electric line of force passing through it.
(B) the particle moves opposite to the electric line of force passing through it.
(C) the direction of acceleration of the particle is tangential to the electric line of force at every instant.
(D) the direction of acceleration of the particle is normal to the electric line of force at every instant.

22. There is a steady water flow in a horizontal tube in which one part has cross sectional area A1 and the
other part has cross sectional area A2. Assume that water is incompressible.
If A1/A2 = 16, the ratio of the speed u1 in part 1 and the speed u2 in part 2, i.e. u1/u2 is :

1 1
(A) (B) 4 (C) (D) 1
16 4

23. Positive point charges of magnitude are placed at all the twelve ‘hour’ positions of a clock of radius r. The
clock is mounted on a wall in the normal way. The charge at the position ‘6’ is removed. The resulting
electric field at the centre of the clock is :

1 q
(A) 0 (B) 4 2 in the horizontal direction.
o r

1 q 1 q
(C) 4 2 vertically upward. (D) 4 2 vertically downward
o r o r

24. The pair of quantities that do not have the same dimensions is :
(A) Latent heat, specific heat
(B) Gravitational force, Coulomb force
(C) Kinetic energy pf a freely falling body, potential energy of a compressed spring
(D) Coefficient of friction, number of molecules in a container.

25. A block of wood is floating on oil with half of its volume submerged. If the density of oil 840 kg m–3, the
relative density of wood (relative to water) is :
(A) 0.84 (B) 0.42 (C) 0.21 (D) 1.00

26. We are given n resistors, each of resistance R. The ratio of the maximum to minimum resistance that can
be obtained by combining them is :
(A) nn (B) n (C) n2 (D) logn

PAGE # 1414
27. A light string goes over a frictionless polley. At its one end hangs a mass of 2 kg and at the other end
hangs a mass of 6 kg. Both the masses are supported by hands to keep them at rest. When the masses
are released, they being to move and the string gets taut.
(Take g = 10 ms–2) The tension in the string during the motion of the masses is :
(A) 60 N (B) 30 N (C) 20 N (D) 40 N

28. The escape speed of a body from the earth regarded as a uniform (non-rotating) sphere depends on :
(A) the mass of the body.
(B) the direction of projection.
(C) the location of the projection on the earth's surface.
(D) the height of location from the earth’s surface from where the body is launched.

29 . On a day during the Monsoon season, the relative humidity at a place is 90% and the temperature is
15°(C) The saturation vapour pressure of water at this temperature is 0.0169 × 105 Pa. The partial pressure
of water vapour in the air on that day is
(A) 0.9 × 105 Pa (B) 0.0169 × 105 Pa (C) 0.0152 x 105 Pa (D) 1.0 x 105 Pa

30. It is now known that a proton and a neutron consist of three elementary units called 'quarks'. Two types

2 1
of quarks are involved: the 'up' quark (u) of charge + e and the ‘down’ quark (d) of charge – e, where
3 3

e is the magnitude of election's charge. u and d are their antiparticles. An antiparticle has the same
magnitude of charge as the particle but of opposite sign. The possible compositions of a proton and a
neutron are :
(A) p : uud ; n: udd
(B) p : uuu ; n: ddd

(C) p : uu d ; n: ud d

(D) p : uu u ; n: ud d

CHEMISTRY

31. The volume of 0.5 M aqueous NaOH solution required to neutralize 10 ml of 2 M aqueous HCl solution is:
(A) 20ml (B) 40ml (C) 80ml (D) 120ml

32. The compound that can be purified by sublimation is :


(A) Ammonium Sulphate (B) Calcium Carbonate
(C) Calcium Oxide (D) Aluminium Chloride

33. Penicillin was discovered by :


(A) Alexander G. Fleming (B) Emil Fisher
(C) Robert B. Woodward (D) van’t Hoff

34. Among butane,1-butene,1-butanol and butanal, the compound which is most polar is
(A) butane (B) 1-butene (C) 1-butanol (D) butanal

PAGE # 1515
35. Among ethanol, dimethyl ether, methanol, and propanal, the isomers are :
(A) ethanol, dimethyl ether, methanol and propanal
(B) ethanol and methanol
(C) ethanol, dimethyl ether, and methanol
(D) ethanol and dimethyl ether

36. Among Li, Be, N and F, the element having the largest atomic radius, is :
(A) Li (B) Be (C) N (D) F

37. The proof of oxidizing action of hydrogen peroxide in acid solution is in the formation of :
(A) O2 (B) H2O
(C) both H2O and O2 (D) both H3O+ and O2

38. A gel toothpaste is a mixture of a :


(A) liquid in a solid (B) solid in a gas (C) liquid in a liquid (D) gas in a solid

39. 3.01×1023 molecules of elemental Sulphur will react with 0.5 mole of oxygen gas completely to produce
(A) 6.02 × 1023 molecules of SO3 (B) 6.02 × 1023 molecules of SO2
(C) 3.01 × 1023 molecules of SO3 (D) 3.01 x 1023 molecules of SO2

40. The pair of metals which will produce hydrogen gas in reaction with acid is :
(A) Mg, Cu (B) Mg, Ag (C) Zn, Pb (D) Cu, Zn

41. The correct order of pH is :


(A) Lemon juice < water < Milk of Magnesia (B) Lemon juice > water > Milk of Magnesia
(C) Water > lemon juice > milk of Magnesia (D) Milk of Magnesia > lemon juice > water

42. The solubility of a gas in a solution is measured in three cases as shown in the figure given below where
w is the weight of a solid slab placed on the top of the cylinder lid. The solubility will follow the order :
w w w w w w

gas gas
gas

solution solution solution

(A) a > b > c (B) a < b < c (C) a = b = c (D) a >b < c

43. A Daniel cell converts :


(A) electrical energy to chemical energy (B) chemical energy to electrical energy
(C) mechanical energy to chemical energy (D) potential energy to electrical energy

PAGE # 1616
44. One of the major components in compressed natural gas (CNG) is :
(A) carbon dioxide (B) acetylene (C) methane (D) water gas

45. Dehydration of 2-pentanol gives :


(A) pentane (B) Only 1-pentene
(C) Only 2-pentene (D) A mixture of 1-pentene and 2-pentene

BIOLOGY
46. A cancer which is not a tumor is :
(A) Lymphoma (B) Leukemia (C) Prostate cancer (D) Oral cancer

47. The phase of the cell cycle in which DNA synthesis takes place is :
(A) G1 phase (B) S phase (C) G2 phase (D) G0 phase

48. You have a tube containing 102 bacteria. You have taken out 102 bacteria. How many bacteria are left in
the tube ?
(A) approximately 107 (B) approximately 106 (C) approximately 105 (D) approximately 109

49. Association in which both the organisms get benefited is :


(A) Commensalism (B) Mutualism (C) Ammensalism (D) Parasitism

50. You are part of a scientific expedition that has ventured deep into the Amazon rain forest. You spot a tree
with branches spread over a large area. What can you conclude about the root structure of the tree?
(A) It is dicotyledonous
(B) It is monocotyledonous
(C) It may be either monocotyledonous or dicotyledonous
(D) There is no correlation between foliage and root structure

51. Alleles are :


(A) Different forms of the same protein (B) Two different genes
(C) Different forms of the same gene (D) Two different proteins

52. Diet rich in carotenoids is beneficial because :


(A) They act as free radical scavengers (B) They absorb harmful UV rays
(C) They help in curing cancer (D) They are rich source of iron

53. The amount of CO2 plant is greater at night than during the day because :
(A) The rate of respiration is higher at night.
(B) More CO2 is produced because it is colder during the night.
(C) Photosynthesis during the day uses up some of the CO2 produced by respiration.
(D) More glucose is available for respiration during the night

54. Osmosis takes place between two solutions separated by a semipermeable membrane because.
(A) Water molecules move from the more dilute solution to the less dilute solution
(B) Solute molecules move from the less dilute solution to the more dilute solution
(C) Water molecules move from the less dilute solution to the more dilute solution
(D) Solute molecules move from the more dilute solution to the less dilute solution

PAGE # 1717
55. Arteries do not have valves but veins do, because :
(A) Arteries have a narrower lumen than veins
(B) Arteries have thicker walls than veins
(C) Arteries carry oxygenated blood whereas veins carry deoxygenated blood
(D) Valves prevent backflow of blood in veins

56. If a person's spinal cord is injured which of the following functions might be affected ?
(A) Talking (B) Seeing (C) Sneezing (D) Hearing

57. Use of an insecticide in an island made a certain insect species extinct. A few years later a tree species
was also found to be extinct. This may be because :
(A) The tree is pollinated by the insect (B) The insect used to live on the tree
(C) The insect used to eat the leaves of the tree (D) The tree is insectivorous

58. Why is it advisable to take antibiotics if someone is suffering from gastric ulcers ?
(A) Antibiotics reduce gastric acid secretion
(B) antibiotics prevent ulcers from being infected
(C) Antibiotics reduce gastric enzyme secretion
(D) Gastric ulcers are caused by bacteria

59. You are synthesizing a 5 amino acid containing peptide from the pool of 20 standard amino acids by
choosing amino acids randomly. What is the probability of selecting Gly–Gly–Gly-Gly
(A) 0 (B) 1/205 (C) 1/20 (D) 1/(20 × 5)

60. The appropriate description of animal hormones is,


(A) Have size smaller than 5 kDa (B) Comprise of proteins only
(C) Comprise of lipid molecule only (D) Are pour directly into the blood

PART-II (2 Mark)
MATHEMATICS

61. Five real numbers x1, x2, x3, x4, x5 are such that :

x1  x 2  x 3  x 4  x 5
x1  1  2 x 2  4  3 x 3  9  4 x 4  16 + 5 x 5  25 =
2

x1  x 2  x 3  x 4  x 5
The value of is
2
(A) not uniquely determined (B) 55
(C) 110 (D) 210

62. The coefficient of x30 in the expansion of :


(1 + 2x + 3x2 + .... 21x20)2 is :
(A) 2706 (B) 2450 (C) 1481 (D) 256

PAGE # 1818
63. A billiards table of grid size 8 × 10 has four holes in the A, B, C, D. A ball starting from the point P on the
side AD, as in the figure, strikes against CD at an angle’ 45°, gets reflected and moves on striking against
the other sides. Which hole will the ball fall into ?
D C

A B

(A) B (B) C (C) D (D) no hole

64. In triangle ABC, the coordinates of A and B are respectively (2,3) and (8,10). It is known that C also
has integer coordinates. The minimum possible area of ABC is :

1 3
(A) 0 (B) (C) 1 (D)
2 2

65. A shop-keeper increases the price of a commodity by x%. some time later, he reduces the new price

1 1
by y% and notices that the price is now the same as it was originally. The value of  is :
y x

1 1
(A)  (B) 0 (C) (D) not uniquely determinable
100 100

PHYSICS

66. An Insulated vessel contains 0.4 kg of warer at 0ºC. A piece of 0.1 kg ice at –15°C is put into it and
steam at 100ºC is bubbled into it until all ice is melted and finally the contents are liquid water at 40ºC.
Assume that the vessel does not give or take any heat and there is no loss of matter and heat to the
surroundings. Specific heat of ice is 2.2 × 103 J kg-1K-1, heat of fusion of water is 333 × 103 J kg–1 and heat
of vaporization of water is 2260 × 103 J kg–1. The amount of steam that was bubbled into the water is
about :
(A) 34.7 gram (B) 236.0 gram (C) 0.023 gram (D) 48.0 gram

67. A light source is placed 100 cm away from a screen. A converging lens placed at a certain position
between the source and the screen focuses the image of the source on the screen. The lens is moved a
distance of 40 cm and it is found that it again focuses the image of the source on the screen. The focal
length of the lens is :
(A) 21 cm (B) 30 cm (C) 40 cm (D) 67 cm

PAGE # 1919
68. A rubber pipe with a diameter of 10 cm is connected to a nozzle 2 cm in diameter. Water flowing through
the. pipe at a speed of 0.6 ms–1, comes out like a jet through the nozzle. The backward force of the
nozzle is about :
(A) 7.7 N (B) 67.9 N (C) zero (D) 2.8 N

69. A power supply of fixed emf E is connected to a variable resistor (resistance = R) and an ammeter of
resistance RA. The internal resistance of the supply is r. The potential difference bctween the terminals of
the power supply in closed circuit is denoted by V and the current in the circuit is denoted by I. A graph
is obtained between V and I by drawing varying amount of current from the supply. The correct V-I graph
for the given situation is :
V V

E
E

E/r I I
(I) (II)

I
I E/r

(III) (IV)

(A) Graph 1 (B) Graph II (C) Graph III (D) Graph IV

70. A thermometer uses ‘density of water’ as thermometric property. The actual reading in the thermometer
is ‘height of water’(h) which is inversely proportional to density of water (d). In a certain temperature
range, density of water varies with temperature as shown. The graph is symmetric about the maximum

d T1 T2
D e nsity of w ate r

Tempeature
Two identical bodies (of same mass and specific heat) at different temperatures T1 and T2 show the same
reading of height h1 = h2 in the thermometer. The bodies are brought into contact and allowed to reach
thermal equilibrium. The thermometer reading ‘height of water for final equilibrium state h1 satisfies.

h1  h2
(A) hf = = h1  h2
2
(B) hf > h1 = h2
(C) hf < h1 = h2
(D) hf may be greater or less than h1 = h2 , depending on the specific heat of the bodies.

PAGE # 2020
CHEMISTRY

71. The density of a salt solution is1.13 g cm–3 and it contains 18% of NaCI by weight. The volume of the
solution containing 36.0 g of the salt will be :
(A) 200 cm3 (B) 217 cm3 (C) 177 cm3 (D) 157cm3

72. Esterification of a compound ‘’X’’ with molecular formula C3H6O2 with an alcohol ‘’Y’’ produces a com-
pound with molecular formula C5H10O2. X and Y, respectively, are :
(A) propanoic acid and methanol (B) propanoic acid and ethanol
(C) acetic acid and ethanol (D) butyric acid and methanol

73. 10 g of a crystalline metal sulphate salt when heated generates approximately 6.4 g of an anhydrous salt
of the same metal. The molecular weight of the anhydrous salt is 160 g. The number of water molecules
present in the crystal is :
(A) 1 (B) 2 (C) 3 (D) 5

74. In the reaction of FeSO4 with K2Cr2O7 in the presence of an acid, the changes in the formal oxidation
numbers of Cr and Fe, respectively, are :
(A) 1 and 5 (B) 3 and 1 (C) 5 and 1 (D) 1 and 3

75. An aqueous solution of a metal salt (X) reacts with carbon dioxide to give a water soluble compound (Y)
which on heating gives the starting metal salt back. Y when present in water does not produce lather with
soap. The compounds X and Y, respectively are
(A) CaCO3 and Ca(HCO3)2 (B) Mg (HCO3)2 and MgCO3
(C) Ca(HCO3)2 and CaCO3 (D) MgCO3 and Mg (HCO3)2

BIOLOGY

76. Plasmolysis is not observed in boiled plant tissue because :


(A) The cell wall becomes impermeable to water
(B) The cell membrane disintegrates
(C) The cell wall disintegrates
(D) The cell membrane becomes impermeable to water

77. If the analogy of a city is applied to a eukaryotic cell then which of the following statements is correct ?
(A) Nucleus as a library and ribosome as a slaughter house
(B) Nucleus as a police station and mitochondria as powerhouse
(C) Mitochondria as powerhouse and Golgi as a cargo sorting facility
(D) Mitochondria as powerhouse and nucleus as slaughter house

PAGE # 2121
78. The placenta tissue in mammals originates from :
(A) The mother only
(B) The foetus only
(C) Both the mother and the foetus
(D) Originates from the mother but taken over by the foetus

79. Both bacteria and amoeba are unicellular organisms which may live in aqueous environment. Bacteria do
not burst by taking up water from the surrounding because they have cell walls. Amoeba do not have cell
walls but yet do not burst because :
(A) Amoeba eliminates excess water by the help of contractile vacuoles.
(B) The cytoplasm of amoeba is isotonic to surrounding water.
(C) The cytoplasm of amoeba is hypotonic to surrounding water.
(D) Amoeba does not take up water from the surroundings.

80. In the lunch, you ate boiled green vegetables, a piece of cooked meat, one boiled egg and a sugar candy.
Which one of these foods may have been digested first ?
(A) Boiled green vegetables (B) The piece of cooked meat
(C) Boiled egg (D) Sugar candy

PAGE # 2222
YEAR-2009 (KVPY-STREAM-SA)

PART-I (1 Mark)
MATHEMATICS

x5
1. The real numbers x satisfying > 1are precisely those which satisfy
1 x
(A) x < 1 (B) 0 < x < 1 (C) – 5 < x < 1 (D) –1 < x < 1

2. Let tn denote the number of integral sided triangle with distinct sides chosen from {1, 2, 3 ———n). Then
t20 – t10 equals
(A) 81 (B) 153 (C) 163 (D) 173

3. The number of pairs of reals (x, y) such that x = x2 + y2 and y = 2xy is


(A) 4 (B) 3 (C) 2 (D) 1

4. How many positive real number x satisfy the equation


x3 – 3 |x| + 2 = 0 ?
(A) 1 (B) 3 (D) 4 (D) 6

5. Let (1 + 2x)20 = a0 + a1x + a2x2 + .... + a20x20 . Then, 3a0 + 2a1 + 3a2 + 2a3 + 3a4 + 2a5 + .... 2a19 + 3a20.
equals to :

5.3 20 – 3 5.3 20  3 5.3 20  1 5.3 20 – 1


(A) (B) (C) (D)
2 2 2 2

6. let P1, P2, P3, P4, P5 be five equally spaced points on the circumfercence of a circle of radius 1, centred
at O. Let R be the set of point in the plane of the circle that are closer to O than any of P1, P2, P3, P4, P5
Then R is a -
(A) circular region (B) rectangular region
(C) pentagonal region (D) oval region that is not circular

7. A company situated at (2,0) in the xy-plane charges Rs. 2 per km for delivery. A second company at (0,3)
chargs Rs. 3 per km for delivery. The region of the plane where it is cheaper to use the first company is -
(A) the inside of the circle (x + 5.4)2 + y2 = 18.72
(B) the outside of the circle (x + 1.6)2 + (y – 5.4)2 = 18.72
(C) the inside of the circle (x – 1.6)2 + (y + 5.4)2 = 18.72
(D) the outside of the circle (x – 5.4)2 + (y + 1.6)2 = 18.72

8. In a right triangle ABC. the in circle touches the hypotenuse AC at D. If AD = 10 and DC = 3, the inradius
of ABC is -
(A) 5 (B) 4 (C) 3 (D) 2

9. The sides of a quadrilateral are all positive integers and three of them are 5, 10, 20. How many possible
value are there for the fourth side ?
(A) 29 (B) 31 (C) 32 (D) 34

PAGE # 2323
10. If the volume of a sphere increases by 72.8%, then its surface area increases by -
(A) 20% (B) 44% (C) 24.3% (D) 48.6%

11. If the decimal 0.d25d25d25 ———is expressible in the form n/27, then d+n must be
(A) 9 (B) 28 (C) 30 (D) 34

12. At what time between 10 O’clock and 11 O’ close are the two hands of clock symmetric with respect to
the vertical line (give the answer to the nearest second) ?
(A) 10h 9m 13s (B) 10h 9m 14s (C) 10h 9m 22s (D) 10h 9m 50s

13. A woman has 10 keys out of which only one opens a lock. She tries the keys one after the another
(keeping aside the failed ones) till she suceeds in opening the lock. What is the chance that it is the
seventh key that works ?

7 1 3 1
(A) (B) (C) (D)
10 2 10 10

14. In a certain school 74% students like circket, 76% students like football and 82% like tennis. Then all the
three sports are liked by at least
(A) 68% (B) 32% (C) 77% (D) 36%

15. Let Sn be the sum of all integers k such that 2n < k < 2n+1, for n  1. Then 9 divides Sn if and only if
(A) n is odd (B) n is of the form 3k+1
(C) n is even (D) n is of the form 3k + 2

PHYSICS

16. A boy standing on the foothpath tosses a ball straight up and catch it. The driver of a car passing by
moving with uniform velocity sees this.

The trajectory of the ball as seen by the driver will be -

(A) (B) (C) (D)

17. Consider two spherical planets of same average density. Planet 2 is 8 times as massive as planet 1.
The ratio of the acceleration due to gravity on the second planet to that on the first is.
(A) 1 (B) 2 (C) 4 (D) 8

PAGE # 2424
18. Two immiscible liquids, A and B are kept in a U-tube. If the density of liquid A is smaller than the density
of liquid B, then the equilibrium situation is.

A A A

(A) (B) (C) (D) None of these

19. In !he figure below a ray of light travelling in a medium of refractive index µ passes through two different
connected rectangular blocks of refractive indices µ1 and µ2 (µ2 > µ1).

µ µ1 µ2

The angle of incidence 1 is increased slightly. The angle 2


(A) increases.
(B) decreases.
(C) remains the same
(D) increases or decreases depending on the value of (µ1/µ2).

20. Two charges of same magnitude move in two circles of radii R1 = R and R2 = 2R in a region of constant

uniform magnetic field B 0 .

The work W 1 and W 2 done by the magnetic field in the Two cases, respectively are such that
(A) W 1 = W 2 = 0 (B) W 1 > W 2 (C) W 1 = W 2  0 (D)W 1 < W 2

21. Two charges +q and –q are placed at a distance b apart as shown in the figure below.
B

P A

C
b/2

+q –q
b
The electric field at a point P on the perpendicular bisector as shown as :

(A) along vector 


A
(B) along vector 
B
(C) along vector 
C
(D) Zero

PAGE # 2525
22. A block of mass M is at rest on a plane surface inclined at an angle  to the horizontal The magnitude of
force exerted by the plane on the block is :
(A) Mg cos (B) Mg sin  (C) Mg tan (D) Mg

23. We are able to squeeze snow and make balls out of it because of -
(A) anomalous behaviour of water. (B) large latent heat of ice.
(C) large specific heat of water. (D) low melting point of ice.

24. Which of the following phenomena can be demonstrated by light. But not with sound waves in an air
column?
(A) Reflection (B) Diffraction
(C) Refraction (D) Polarziation

25. The temperature of a metal coin is increased by 100°C and its diameter increases by 0.15%. Its area
increases by nearly
(A) 0.15% (B) 0.60% (C) 0.30% (D) 0.0225%

26. The note "Saa" on the Sarod and the Sitar have the same pitch. The property of sound that is most
important in distinguishing between the two instruments is
(A) fundamental frequency (B) intensity
(C) displacement amplitude (D) waveform

235 207
27. 92 U atom disintegrates to 82 Pb with a half-life of 109 years. In the process it emits 7 alpha particles

and n  – particles. Here n is -


(A) 7 (B) 3 (C) 4 (D) 14

28. Consider the circuit below. The bulb will light up of :


S1

S2

~ S3

(A) S1 S2 and S3 are all closed.


(B) S1 is closed but S2 and S3 are open.
(C) S2 and S3 are closed but S1 is open.
(D) S1 and S3 are closed but S2 is open.

PAGE # 2626
29. Two bullbs, one of 200W and the other of 100W, are connected in series with a 100 V battery which has
no internal resistance. Then,

100V

200W 100W

(A) the current passing through the 200W bulb is more than that through the 100W bulb.
(B) the power dissipation in the 200W bulb is more lhan that In the 100 W bulb.
(C) the voltage drop across the 200W bulb is more than that across the 100W bulb.
(D) the power dissipation In the 100W bulb is more than that in the 200W bulb.

30. A solid cube and a solid sphere of identical material and equal masses are heated to the same tempera-
ture and left to cool in the same surroundings. Then
(A) the cube will cool faster because of its sharp edges.
(B) the cube will cool faster because it has a larger surface area.
(C) the sphere will cool faster because it is smooth.
(D) the sphere will cool faster because it has a larger surface area.

CHEMISTRY
31. The element X which forms a stable product of the type XCI4 is -
(A) AI (B) Na (C) Ca (D) Si

32. A mixture of NH4CI and NaCI can be separated by -


(A) filtration (B) Distillation (C) Sublimation (D) Decantation

33. The pair in which the first compound is ionic and the second compound is covalent, is -
(A) Fe(OH)2,CH3OH (B) CH3OH, CH3CH2OH (C) Fe(OH)2, Cu(OH)2 (D) Ca(OH)2, Cu(OH)2

34. In the reaction SO2 + 2H2S  3S + 2H2O, the substance that is oxidized is -
(A) SO2 (B) H2O (C) S (D) H2S

35. Sodium oxide dissolves in water to give sodium hydroxide which indicates its -
(A) acidic character (B) basic character
(C) amphoteric character (D) ionic character

36. For an ideal gas, Boyle’s law is best described by -

P P P P

(A) (B) (C) (D)

T V V T

PAGE # 2727
37. The pH values of
(i) 0.1 M HCl aq (ii) 0.1 M KOH
(iii) tomato juice and (iv) pure water
follow the order -
(A) (i) < (iii) < (iv) < (ii) (B) (iii) < (i) < (iv) < (ii) (C) (i) < (ii) < (iii) < (iv) (D) (iv) < (iii) < (ii) < (i)

38. When calcium carbide is added to water, the gas that is evolved is -
(A) carbon dioxide (B) hydrogen (C) acetylene (D) methane

39. Atomic radii of alkali metals follow the order -


(A) Li > Na > K > Cs (B) K > Cs > Li > Na (C) Na > K > Cs > Li (D) Cs > K > Na > Li

40. The number of possible structural isomers of C3H4 is :


(A) 1 (B) 2 (C) 3 (D) 4

41. Among the four compounds (i) acetone, (ii) propanol, (iii) methyl acetate and (iv) propionic acid, the two
that are isomeric are -
(A) methyl acetate and acetone (B) methyl acetate and propanol
(C) propionic acid and methyl acetate (D) propionic acid and acetone

42. One mole of nitrogen gas on reaction with 3.01 x 1023 molecules of hydrogen gas produces -
(A) one mole of ammonia (B) 2.0 x 1023 molecules of ammonia
(C) 2 moles of ammonia (D) 3.01 × 1023 molecules of ammonia

43. Saponification is -
(A) hydrolysis of an ester (B) hydrolysis of an amide
(C) hydrolysis of an ether (D) hydrolysis of an acid chloride

44. A concentrated solution of lead nitrate in water can be stored in -


(A) an iron vessel (B) a copper vessel
(C) a zinc vessel (D) a magnesium vessel

45. Solubility
g/I
250
KNO3
200
150
KCl
100

50

20 40 60 80 100
Temperature (ºC)

Given the solubility curves of KNO3 and KCl, which of the following statements is not true?
(A) At room temperature the solubility of KNO3 and KCI are not equal.
(B) The solubilities of both KNO3 and KCI increase with temperature.
(C) The solubility of KCI decreases with temperature.
(D) The solubility of KNO3 increases much more as compared to that of KCl with increase in temperature.

PAGE # 2828
BIOLOGY

46. Which one of the following is the smallest in size ?


(A) Bacteria (B) Mitochondrion
(C) Mammalian cell (D) Virus

47. If birds are moved from 30°C to 10ºC, their body temperature :
(A) changes from 30ºC to 10°C (B) increases by 10ºC
(C) does not changes at all (D) decreases by 10ºC

48. Ascorbic acid is a/an.


(A) Strong inorganic acid (B) Hormone
(C) Vitamin (D) Enzyme

49. Bile salts :


(A) break down polypeptide chains (B) emulsify fats and solubilize them
(C) digest fats (D) help breakdown of polysaccharides

50. Dietary fibers are composed of :


(A) Cellulose (B) Amylase (C) Proteins (D) Unsaturated fats

51. ‘On the origin of species , by means of Natural selection’ was written by.
(A) Hugo de vires (B) Charles Darwin (C) Charles dickons (D) Alfred Russell wallace

52. Unlike humans, dogs cannot perspire to get rid to excess metabolic heat. They lose metabolic heat by:
(A) Panting (B) running in windy conditions
(C) taking a bath (D) rolling in the mud

53. Haemodialysis is a treatement option for patients with malfunctions of :


(A) Kidney (B) Heart (C) Liver (D) Lungs

54. An individual has O blood group if his/her blood sample.


(A) Clumps only when antiserum A is added
(B) Clumps only when antiserum B is added
(C) Clumps when both antiserum A and antiserum B are added
(D) Does not clump when either antiserum A or antiserum B is added

55. In warmer, weather, curds from milk forms faster because :


(A) bacteria diffuse better in warmer milk (B) the rate of bacterial multiplication increases
(C) lactogen is better dissolved (D) it is easier to separate protein from water

PAGE # 2929
56. Seedlings grown in dark are :
(A) similar to those grow in light (B) taller than those grow in light
(C) shorter than those grow in light (D) they don’t grow at all

57. In humans, Rhesus conditions can arise when :


(A) father is Rh+ and mother is Rh– (B) father is Rh– and mother is Rh+
(C) either father or mother is Rh+ (D) either father or mother is Rh–

58. The part of the human brain that governs memory and intelligence is :
(A) Cerebrum (B) Hypothalamus (C) Medulla (D) Cerebellum

59. Saturated dietary fats increase the risk of heart disease by :


(A) widening arteries by thinning their walls (B) narrowing veins by carbohydrate deposition
(C) narrowing arteries by fat deposition (D) narrowing arteries by carbohydrate deposition

60. Rotation of crops is carried out to :


(A) increase variation in the mineral content of the soil
(B) increase diversity of plant habitats
(C) increase in nitrogen content of the soil
(D) increase convenience for the farmer

PART-II (2 Marks)
MATHEMATICS

61. Let logab = 4, logcd = 2 where a, b, c, d are natural numbers. Given that b – d = 7, then value of c – a is
(A) 1 (B) – 1 (C) 2 (D) – 2

62. Let P(x) = 1 + x + x2 + x3 + x4 + x5. What is the remainder when P(x12) is divided by P(x) ?
(A) 0 (B) 6
(C) 1 + x (D) 1 + x + x2 + x3 + x4

63. In a triangle ABC the altitudes from B and C on to the opposite sides are not shorter than their respective
opposite sides. Then one of the angles of ABC is -
(A) 30º (B) 45º
(C) 60º (D) 72º

64. In a triangle ABC, AB = AC = 37. Let D be a point on BC such that BD = 7. AD = 33. The length of CD is-
(A) 7 (B) 11 (C) 40 (D) not determinate

65. A line segment  of length a cm is rotated about a vertical line L, keeping the line  in one of the following
three position (I)  is parallel to L and is at a distance of r cm. from L ; (II)  is perpendicular to L and its
mid point is at a distance r cm, from L ; (III)  and L are in the same plane and is inclined to L at an angle
30º with it mid point at a distance r cm. from L. Let A1, A2 , A3 be the areas so generated. If r > (a/2), then
(A) A1 < A3 < A2 (B) A1 = A3 < A2 (C) A2 < A1 < A3 (D) A1 = A2 = A3

PAGE # 3030
PHYSICS

66. A spring balance A reads 2 kg when a block of mass m suspended from it. Anolher balance B reads 3 kg
when a beaker with a liquid is put on its pan. The two balanes are now so arranged that the hanging mass
m is fully immersed inside the liquid in tbe beaker as shown in the figure. In this situattion.

(A) the balance A will read 2 kg and B will read 5 kg.


(B) the balance A will read 2 kg and B will read 3 kg.
(C) the balance A will read less than 2 kg and B will read between 3 kgand 5 kg.
(D) the balance A will read less than 2 kg and B will read 3 kg.

67. According to the quantum theory, a photon of electromagnetic radiation of frequency  has energy
E = h where h is known as planck’s constant. According to the theory of relativity, a particle of mass m
has equivalent energy E = mc2, where c is speed of light. Thus a photon can be treated as a particle

h
having effective mass m = . If a flash of light is sent horizonatally in earth’s gravitational field, then
C2
photons while traveling a horizontal distance d would fall through a distance given by -

gd2 h mcd2
(A) (B) (C) (D) zero
2c 2 mc h

68. A solid square plate is spun around different axes with the same angular speed. In which of the following
choice of axis of rotation will the kinetic energy of the plate be the largest ?
(A) through the central normal to the plate. (B) along one of the diagonals of the plate.
(C) along one of the edges of the plate. (D) through one corner normal to the plate.

69. An object is placed 0.40 m from one of the two lenses L1 and L2 of focal lengths 0.20 m and 0.10 m
respectively as depicted in the figure. The separation between the lenses is 0.30 m.
L1 L2

0.40m + 0.30m

The final image formed by this two lens system is at


(A) 0.13 m to the right of the second lens. (B) 0.05 m to the right of the second lens.
(C) 0.13 m to the left of the second lens (D) Infinity

PAGE # 3131
70. 5 charges each of magnitude 10–5 C and mass 1 kg are placed (fixed) symmetrically about a movable
central charge of magnitude 5 × 10–5C and mass 0.5 kg as shown. The charge at P1 is removed. The
acceleration of the central charge is-
P1

P2 P5
O

P3 P4
1
[Given OP1 = OP2 = OP3 = OP4 = OP5 = 1 m ; 4 = 9 × 109 in SI units]
0

(A) 9 m s–2 upwards (B) 4.5 m s–2 upwards (C) 9 m s–2 downwards (D) 4.5 m s–2 downwards

CHEMISTRY
71. Reaction of NaCl with conc. H2SO4 liberates a gas X that turns moist blue litmus paper red. When gas X
is passed into a test tube containing egg shell powder suspended in water another gas, Y is generated
which when passed through lime water makes it milky. The gases X and Y respectively, are -
(A) HCl and CO2 (B) Cl2 and CO2 (C) SO2 and CO2 (D) SO2 and HCl

72. 10 ml of an aqeuous solution containing 222 mg of calcium chloride (mol. wt. = 111) is diluted to 100 ml.
The concentration of chloride ion in the resulting solution is -
(A) 0.02 mol/lit. (B) 0.01 mol/lit. (C) 0.04 mol/lit (D) 2.0 mol/lit.

73. Aluminium reduces maganese dioxide to manganese at high temperature. The amount of aluminium
required to reduce one gram mole of manganese dioxide is -
(A) 1/2 gram mole (B) 1 gram mole (C) 3/4 gram mole (D) 4/3 gram mole

74. Ethanol on reaction with alkaline KMnO4 gives X which when reacted with methanol in the presence of an
acid gives a sweet smelling compound. Y. X and Y respectively, are -
(A) acetaldehyde and acetone (B) acetic acid and methyl acetate
(C) formic acid and methyl formate (D) ethylene and ethyl methyl ether

75. The pH of a 10 ml aqueous solution of HCl is 4. The amount of water to be added to this solution in order
to change its pH from 4 to 5 is -
(A) 30 ml (B) 60 ml (C) 90ml (D) 120 ml

BIOLOGY
76. Proteins are synthesized on :
(A) Cytoskeleton (B) Mitochondria (C) Ribosomes (D) Golgi apparatus

77. Which of the following allows light to focus in visual perception ?


(A) Retina (B) Retinal pigment (C) Iris (D) Cornea

78. During cell division if there is one round of chromosome duplication followed by one round of cell division
, the number of chromosomes the daughter cells will have as compared to the mother is :
(A) equal (B) double (C) half (D) one fourth

79. Similar type of vegetation can be observed, in the same :


(A) latitude (B) longitude (C) country (D) continent

80. Which of the following ecological food chain does not represent an erect pyramid of numbers ?
(A) Grass Rodent Snake (B) Tree-Bird-Avian parastie
(C) Grass-Deer-Tiger (D) Insect-Chicken-Human

PAGE # 3232
YEAR-2010 (KVPY-STREAM-SA)

PART-A (1 Mark)
MATHEMATICS
1. A student notices that the roots of the equation x2 + bx + a = 0 are each 1 less than the roots of the
equation x2 + ax + b = 0. Then a + b is :
(A) possibly any real number (B) – 2
(C) – 4 (D) – 5

2. If x, y are real numbers such that


x x
1 1
y – y = 24,
3 3
then the value of (x + y) / (x – y) is :
(A) 0 (B) 1 (C) 2 (D) 3

12  2 2  3 2  .....  n 2
3. The number of positive integers n in the set {1, 2, 3, ...., 100} for which the number
1  2  3  ......  n
is an integer is :
(A) 33 (B) 34 (C) 50 (D) 100

4. The three different face diagonals of a cuboid (rectangular parallelopiped) have lengths 39, 40, 41. The
length of the main diagonal of the cuboid which joins a pair of opposite corners :
(A) 49 (B) 49 2 (C) 60 (D) 60 2

5. The sides of a triangle ABC are positive integers. The smallest side has length . What of the following
statements is true ?
(A) The area of ABC is always a rational number.
(B) The area of ABC is always an irrational number.
(C) The perimeter of ABC is an even integer.
(D) The information provided is not sufficient to conclude any of the statements A, B or C above.

6. Consider a square ABCD of Side 12 and let M, N be the midpoints of AB, CD respectively. Take a point P
on MN and let AP = r, PC = s. Then the area of the triangle whose sides are r, s, 12 is :

rs rs
(A) 72 (B) 36 (C) (D)
2 4

7. A cow is tied to a corner (vertex) of a regular hexagonal fenced area of side a meters by a rope of length
5a / 2 meters in a grass field. (The cow cannot graze inside the fenced area.) What is the maximum
possible area of the grass field to which the cow has access to graze ?

5
(A*) 5 a2 (B) a2 (C) 6a2 (D) 3a2
2

PAGE # 33
8. A closed conical vessel is filled with water fully and is placed with its vertex down. The water is let out at
a constant speed. After 21 minutes, it was found that the height of the water column is half of the original
height. How much more time in minutes does it empty the vessel ?
(A) 21 (B) 14 (C) 7 (D) 3

9. I carried 1000 kg of watermelon in summer by train. In the beginning, the water content was 99%. By the
time I reached the destination, the water content had dropped to 98%. The reduction in the weight of the
watermelon was:
(A) 10 kg (B) 50 kg (C) 100 kg (D) 500 kg

10. A rectangle is divided into 16 sub-rectangles as in the figure; the number in each sub-rectangle repre-
sents the area of that sub-rectangle. What is the area of the rectangle KLMN ?

(A) 20 (B) 30 (C) 40 (D) 50

PHYSICS

11. A hollow pendulum bob filled with water has a small hole at the bottom through which water escapes at a
constant rate. Which of the following statements describes the variation of the time period (T) of the
pendulum as the water flows out ?
(A) T decreases first and then increases. (B) T increases first and then decreases.
(C) T increases throughout. (D) T does not change.

12. A block of mass M rests on a rough horizontal table. A steadily increasing horizontal force is applied such
that the block starts to slide on the table without toppling. The force is continued even after sliding has
started. Assume the coefficients of static and kinetic friction between the table and the block to be equal.
The correct representation of the variation of the frictional forces, ƒ, exerted by the table on the block with
time t is given by :

(A) (B) (C) (D)

13. A soldier with a machine gun, falling from an airplane gets detached from his parachute. He is able to
resist the downward acceleration if he shoots 40 bullets a second at the speed of 500 m/s. If the weight
of a bullet is 49 gm, what is the weight of the man with the gun ? Ignore resistance due to air and assume
the acceleration due to gravity g = 9.8 m/s2
(A) 50 kg (B) 75 kg (C) 100 kg (D) 125 kg

PAGE # 34
14. A planet of mass m is moving around a star of mass M and radius R in a circular orbit of radius r. The star
abruptly shrinks to half its radius without any loss of mass. What change will be there in the orbit of the
planet ?
(A) The planet will escape from the star. (B) The radius of the orbit will increase.
(C) The radius of the orbit will decrease. (D) The radius of the orbit will not change.

15. Figure (a) below shows a Wheatstone bridge in which P, Q, R, S are fixed resistances, G is a galvanom-
eter and B is a battery. For this particular case the galvanometer shows zero deflection. Now, only the
positions of B and G are interchanged,. as shown in figure (b). The new deflection of the galvanometer.

(A) is to the left. (B) is to the right.


(C) is zero. (D) depends on the values of P, Q, R, S

16. 12 positive charges of magnitude q are placed on a circle of radius R in a manner that they are equally
spaced. A charge +Q is placed at the centre. If one of the charges q is removed, then the force on Q is :
(A) zero

qQ
(B) 4 R2 away from the position of the removed charge.
0

11qQ
(C) 4 R2 away from the position of the removed charge.
0

qQ
(D) 4 R2 towards the position of the removed charge.
0

17. An electric heater consists of a nichrome coil and runs under 220 V, consuming 1 kW power. Part of its
coil burned out and it was reconnected after cutting off the burnt portion. The power it will consume now
is :
(A) more than 1 kW. (B) less that 1 kW, but not zero.
(C) 1 kW. (D) 0 kW.

18. White light is split into a spectrum by a prism and it is seen on a screen. If we put another identical
inverted prism behind it in contact, what will be seen on the screen ?
(A) Violet will appear where red was
(B) The spectrum will remain the same
(C) There will be no spectrum, but only the original light with no deviation.
(D) There will be no spectrum, but the original light will be laterally displaced.

PAGE # 35
19. Two identical blocks of metal are at 20ºC and 80ºC, respectively. The specific heat of the material of the
two blocks increases with temperature. Which of the following is true about the final temperature Tƒ when
the two blocks are brought into contact (assuming that no heat is lost to the surroundings) ?
(A) Tƒ will be 50ºC.
(B) Tƒ will be more than 50ºC.
(C) Tƒ will be less than 50ºC.
(D) Tƒ can be either more than or less than 50ºC depending on the precise variation of the specific heat
with temperature.

20. A new temperature scale uses X as a unit of temperature, where the numerical value of the temperature
tX in this scale is related to the absolute temperature T by tX = 3T + 300. If the specific heat of a material
using this unit is 1400 J kg–1 X–1 its specific heat in the S.I. system of units is :
(A) 4200 J kg–1 K–1
(B) 1400 J kg–1 K–1
(C) 466.7 J kg–1 K–1
(D) impossible to determine from the information provided

CHEMISTRY

21. The boiling points of 0.01 M aqueous solutions of sucrose,NaCl and CaCl2 would be :
(A) the same (B) highest for sucrose solution
(C) highest for NaCl solution (D) highest for CaCl2 solution

22. The correct electronic configuration for the ground state of silicon (atomic number 14) is :
(A) 1s2 2s2 2p6 3s2 3p2 (B) 1s2 2s2 2p6 3p4 (C) 1s2 2s2 2p4 3s2 3p4 (D) 1s2 2s2 2p6 3s1 3p3

23. The molar mass of CaCO3 is 100 g. The maximum amount of carbon dioxide that can be liberated on
heating 25 g of CaCO3 is :
(A) 11 g (B) 5.5 g (C) 22 g (D) 2.2 g

24. The atomic radii of the elements across the second period of the periodic table.
(A) decrease due to increase in atomic number
(B) decrease due to increase in effective nuclear charge
(C) decrease due to increase in atomic weights
(D) increase due to increase in the effective nuclear charge

25. Among NH3, BCl3, Cl2 and N2, the compound that does not satisfy the octet rule is :
(A) NH3 (B) BCl3 (C) Cl2 (D) N2

26. The gas produce on heating MnO2 with conc. HCl is


(A) Cl2 (B) H2 (C) O2 (D) O3

27. The number of covalent bonds in C4H7Br, is :

PAGE # 36
(A) 12 (B) 10 (C) 13 (D) 11
28. An aqueous solution of HCl has a pH of 2.0. When water is added to increase the pH to 5.0, the hydrogen
ion concentration :
(A) remains the same (B) decreases three-fold
(C) increases three-fold (D) decreases thousand-fold

29. Consider two sealed jars of equal volume. One contains 2 g of hydrogen at 200 K and the other contains
28 g of nitrogen at 400 K. The gases in the two jars will have :
(A) the same pressure. (B) the same average kinetic energy.
(C) the same number of molecules. (D) the same average molecular speed.

30. Identify the stereoisomeric pair from the following choices.


(A) CH3CH2CH2OH and CH3CH2OCH3 (B) CH3CH2CH2Cl and CH3CHClCH3

(C) and (D) and

BIOLOGY

31. Which of the following is a water-borne disease ?


(A) Tuberculosis (B) Malaria (C) Chickenpox (D) Cholera

32. In has seminal work on genetics, Gregor Mendel described the physical traits in the pea plant as being
controlled by two 'factors'. What term is used to define these factors today ?
(A) Chromosomes (B) Genes (C) Alleles (D) Hybrids

33. A majority of the tree species of Peninsular Indian origin fruit in the months of :
(A) April - May (B) August - September
(C) December - January (D) All months of the year

34. In frogs, body proportions do not change with their growth. A frog that is twice as long as another will be
heavier by approximately.
(A) Two-fold (B) Four-fold (C) Six-fold (D) Eight-fold

35. Which of the following has the widest angle of binocular vision ?
(A) Rat (B) Duck (C) Eagle (D) Owl

36. The two alleles of a locus which an offspring receives from the male and female gametes are situated on:
(A) Two different homologs of the same chromosome.
(B) Two different chromosomes.
(C) Sex chromosomes.
(D) A single chromosome.

37. Ants locate sucrose by :


(A) Using a strong sense of smell.
(B) Using a keen sense of vision.
(C) Physical contact with sucrose.
(D) Sensing the particular wavelength of light emitted / reflected by sucrose.

PAGE # 37
38. The interior of a cow-dung pile kept for a few days is quite warm. This is mostly because :
(A) Cellulose present in the dung is a good insulator.
(B) Bacterial metabolism inside the dung releases heat.
(C) Undigested material releases heat due to oxidation by air.
(D) Dung is dark and absorbs a lot of heat.

39. Which one of these is the correct path for a reflex action ?
(A) Receptor-Motor Neuron-Spinal Cord-Sensory Neuron-Effector.
(B) Effector-Sensory Neuron-Spinal Cord-Motor Neuron-Receptor.
(C) Receptor-Sensory Neuron-Spinal Cord-Motor Neuron-Effector.
(D) Sensory Neuron-Receptor-Motor Neuron-Spinal Cord-Effector.

40. Insectivorous plants digest insects to get an essential nutrient. Other plants generally get this nutrient
from the soil. What is this nutrient ?
(A) Oxygen (B) Nitrogen (C) Carbon dioxide (D) Phosphates

DESCRIPTIVE TYPE QUESTIONS

DISCLAIMER

The subjective problems are based memory rotations of our students hence they may differ slightly
then the original questions. The solutions presented here are in accordance with the given problems
statement.

PART-B (5 Marks)
MATHEMATICS

1. Leela and madan collectea their CD's and sold them such that each C.D. was sold at the same amount
as number of C.D's. From this amount colluted leela juist borrowed Rs. 10, then madan borrowed Rs. 10,
they borrowed money alternately until amount less than Rs. 10 was left for Madan to borrow. Find has
much was left for Madan to borrowed at the end. (5 marks)

2. In ABC, DE is drawn parallel to BC, D on A and B on E. Such that ADE has area 3. Now BE and DC
are joined to get intersection point P. DPE = 1 sq. unit. Find area of ABC ? (5 marks)

3. (i) If there is natural number n relative prime with 10. Then show that there exist another natural number m
such that all digits are 1's and m is div. by 'n'. (5 marks)

a
(ii) Show that every natural number can represented as , where a, b, c  N
10 (10 c  1)
b

PAGE # 38
PHYSICS

4. There is a smooth fixed concave surface. A particle is released from p. Find : (5 marks)

(i) PE as function of 
(ii) KE as a function 
(iii) time taken from P to 
(iv) the reaction force at 

5. (5 marks)

Given: RA << R << RV

V
Restimated =

R = |R – Rest|

(i) Calculate RP

(ii) Calculate RQ

(iii) Find the value of R for which RP = RQ

6. An object is placed. 20 cm from a concave lens of focal length 10 cm : (5 marks)


(i) find the position of the image
(ii) find the position of another concave mirror of focal length 5 cm. where it should be placed right side of
above concave lens such that final image coincides with the object.
(iii) If a plane mirror is placed in place of concave mirror at the same position, then find the position of final
image.

PAGE # 39
CHEMISTRY
7. There are four bottles 1,2,3,4 containing following compounds. (5 marks)

Bottle-1 : ; Bottle-2 :

Bottle-3 : ; Bottle-4 :

Identify the bottle :


(I) (A) whose content does not react with 1N HCl or 1N NaOH.
(B) whose content reacts with 1N NaOH only.
(C) whose content reacts with 1N NaOH and 1N HCl both.
(D) whose content reacts with 1N HCl only.
(II) The bottle whose content is highly soluble in distilled water.

8. A copper ore was treated with HNO3 to form Cu(NO3)2, Which on further reaction with K, forms Cu2,
which decomposes to form Cu22 and 2, 2 was titrated with Na2S2O3. (5 marks)
(I) Balance the reactions involved (find out values of a to )
(a) a Cu + b HNO3  c Cu(NO3)2 + d NO + e H2O
(b) f Cu2  g Cu22 + h 2
(c) i Na2S2O3 + j 2  k Na2S4O6 +  Na

(II) 2.54 g of 2 was evolved : Find the percentage purity of copper in 2 g of ore.

9. Human being required 2500 Kcal of energy per day (5 marks)


C12H22O11 + 12 O2  12 CO2 + 11 H2O
H = – 5.6 × 106 J / mol
(i) Human being required ..................... kJ of energy per day.
(ii) Amount of sucrose required per day and volume of CO2 evolved during the process [at STP]

BIOLOGY
10. Mohini, a resident of Chandigarh went to Shimla with her parents. There she found the same plant that
they have in their backyard, at home, However, she observed that while the plants in their backyard bore
white flowers, those in Shimla had pink flowers. She brought home some seeds of the plant from Shimla
and planted them in Chandigarh. Upon performing self-breeding for several generations she found that the
plant from Shimla produced only white flowers.
(a) According to you what might be the reason for this observation – genetic or environmental factors?
(b) Suggest a simple experiment to determine whether this variation is genetic in nature.
(c) Suggest another experiment to check whether this variation in flower color is due to environmental
factors. (5 marks)

11. The break-down of glucose in a cell occurs in any of the following pathways :

CO2 + H2O (In the presence of O2 e.g.in mitrochondria )

Glucose Pyruvic Ethanol + CO2 (In the absence of O2 e.g.Yeast )


acid
Lactic acid (In the absence of O2 e..g. lactic acid bacteria)

PAGE # 40
Three experiments (A, B, C) have been set up. In each experiment, a flask contains the organism in
growth medium, glucose and a brown dye that changes its colours to yellow when the pH decreases. The
mouth of the flask is attached to a test tube containing lime water (Calcium Hydroxide, as shown in the
figure). in C, but not in A and B, air is removed from the flask before beginning the experiment.

– – ––
– – ––– Lime
– – water

Organism in Culture Medium
+ Glucose + Dye

After a period of growth, the following observations were made :


A : Lime water turns milky ; the dye colour remains the same.
B : The dye colour changes ; lime water does not turn milky.
C : Lime water turns milky ; the dye colour remains the same.

(a) Question : Identify which of the reactions is the pathways depicted above is taking place in each
experiment. Give reasons for your answer. (4 marks)
(b) Question : Identify which of the reactions in the pathways depicted above is expected to occur in Red
Blood Cells (RBCs) (1 marks)

12. A scientist has a house just beside a busy highway. He collects leaves from some plants growing in his
garden to do radio-carbon dating (to estimate the age of the plant by estimating the amount of a radioisotope
of carbon in its tissues). Surprisingly the radio-carbon dating shows that the plant is a few thousand years
old)
(a) Was the result of the radio-carbon dating wrong or can you propose a reason for such an observation?
(3 marks)
(b) What simple experiment can be done to test the reason that you have proposed ? (2 marks)

PAGE # 41
HINTS & SOLUTIONS (YEAR-2007)

ANSWER KEY
Ques. 1 2 3 4 5 6 7 8 9 10 11 12 13 14 15
Ans. B C D C B D D B A B D C D D B
Ques. 16 17 18 19 20 21 22 23 24 25 26 27 28 29 30
Ans. B C B C C B C C A C A C B A C
Ques. 31 32 33 34 35 36 37 38 39 40 41 42 43 44 45
Ans. B A C C D B A B B D A C C A D
Ques. 46 47 48 49 50 51 52 53 54 55 56 57 58 59 60
Ans. A A A D B A D A B D D D D B B
Ques. 61 62 63 64 65 66 67 68 69 70 71 72 73 74 75
Ans. C A B A C A A D A B D D A C C

PART-I (1 Mark)
MATHEMATICS

n n
1. s1(n) = [2.8 + (n – 1) 4] s2(n) = [2.17 + (n – 1)2]
2 2
s1(n) = s2(n)
n n
[2.8 + (n – 1) 4] = [2.17 + (n – 1)2]
2 2
2(n – 1) = 18
n–1=9
n = 10
s1(10) = 5[16 + 36] = 260 = s2(10)

2.

(sin(2x))4 = 1/8
Range  x  [0, 2]
Let 2x = y
given range 0  x  2
0  2x  4
 0  y  4
(sin(2x))4 = 1/8
From above graph we can say,
Total Eight solution
3.r 2 – 8.r  5
3. 0
4.r 2 – 3.r  7
 3r2 – 3r – 5r + 5 > 0 and 4r2 – 3r + 7 > 0 as D<0
 3r(r – 1) –5(r – 1) > 0
 (3r – 5)(r – 1) > 0
(3r – 5)(r – 1) > 0
r  (– , 1)  (5/3, )

42
PAGE # 4242
4. a : (b + c) = b : (c + a)
add 1 both side

abc abc
=
bc ca

a+b+c=0

3/2
3x
5. xx = 2
x
Case - I when base x = 1
Case - II when base x  1

3x
then x3/2 =
2

 3
 x x   = 0
 2

9
 x0x=
4

Hence two solutions

40
 n2 
6. M   
 2 
n 1 

Splitng n in odd number and in even number

20 19
 (2x ) 2   (2y  1)2 
M    +   
 2 
x 1  y 0
2 

20 19
 2 2 1
M  2x  +  2y
x 1 y 0
 2y  
2

20 19

=  2x 2 +  2y 2
 2y
x 1 y 0

20 19
2  20(20  1)( 41)
Now  2x 2  and  (2y 2
 2y ) = 2  19  (19  1)(39 ) + 2  19  20
x 1
6 y 0 6 2

 20  21 41  (19  20  39 ) (19  20)


2x   + 2x +2
 6  6 2

5740 + 4940 + 380 = 11060

43
PAGE # 4343
7. f(x) = x3 + ax2 + bx + c
Since f(x) has three roots when c = 0

 graph can be O

Now, graph of

f(x) = x3 + ax2 + bx + c, when c > 0 can be O

So option (D) is right

8. Case - I
Here a = 1

Case – II

3
Altitute a=1
2
2
a= .
3

9. Let AB = a then BE = a tan


CE
= tan
CF
CF = a cot – a
Now, In GHF
HF 1  tan 
tan = =
GH 2  cot 
Solving we get
2
tan =
3
2
 sin =
13
10. R = 5, c = 6, b = 6
12  x
S= ,= s( s – a)( s – b)( s – c )
2

12  x x x 12 – x
= . . .
2 2 2 2
abc
R=
4

44
PAGE # 4444
36 x
 5 
4 (12  x )x 2 (12 – x )
4
36
 (12)2 – x 2 =
5
2
 36 
 (12) –   = x2
2
 5 
 x = 48/5

11. Let AP = x1, PQ = x2


ar( ADE) 1
Now, =
ar( ABC) 2
2
x1 1
 2 =
( x1  x 2 ) 2
 2x12 = x12 + x22 + 2x1x2
 x12 – x22 – 2x1x2 = 0
x1 2 44
 x2 = 2
x1 2 1
 x2 = 1

1
12. cos2  + cos2  = 3/2 and sin  sin  =
4
1  cos 2 1  cos 2 1
+ = 3/2 and 2sin  sin  =
2 2 2
1
cos 2 + cos 2 = 1 and cos ( – ) – cos ( + ) = ...(ii)
2
2 cos ( + ) cos ( – ) = 1 ...(i)
From (i) and (ii)
cos ( + ) cos ( – ) = cos ( – ) – cos ( + )
1
 cos ( + ) = and cos ( – ) = 1
2

+=
3

1 3x2
13. Area = [x + 2x + 3x] =
2 4

3x2
 3x =
4
 x= 4 3

 Area = 12 3

45
PAGE # 4545
14. Let wealth A, B, C are x, y, z
Given : A = B + C
x=y+z ....(i)
A distribute half of his wealth to B & C
x 2 1
   2 : 1 i.e. &
2 3 3
x
Now, A =
2
2 x
B = y + 2
3  
1 x
C=z+  
3 2
Now, B = A + C
x x x
y+ = +z+
3 2 6
x
y–z=
3
3y – 3z = x ...(ii)
1 2
Let t be fraction that A should distribute and the ratio of distribution is 1 : 2 i.e. < .
3 3
Now, A = (1 – t)x
tx
B=y+
3

2tx
C=z+
3

2tx tx
z+ =y+ + (1 – t) x
3 3

tx
z+ = y + x – tx
3
3z + tx = 3y + 3x – 3tx
4tx = 3 (y – z) + 3x
4tx = 3x + x
4tx = 4x
t = 1.
So, A would distribute his whole wealth to B and C.
Fraction is 1.

15. Put 3 times water of 110 ml to container and take 13 times 25 ml water from container then container
has 5 ml water.

16. Distance = speed × time


Distance covered in car = 50 × 4 = 200 km
He move for 1 hr = 20 km
Dis tan ce 200
Time take while retraining from town to village time = = = 5 hr
Speed 40
Total journey = 200 + 200 + 20
Total time taken = 4 + 1 + 5 = 10
420
average speed = = 42 km/hr
10

46
PAGE # 4646
17. Given 4 × 4 × 4 cubes is mode faces 64
1 × 1 × 1 cubes
total cubes = 64, white = 20, Red = 44
To find minimum number of visible white box
Counting total visible faces of unit cube
Total number of faces of small cube on bigger cube
except boundry cubes = 4 × 6 = 24
Counting boundry cube = 16 + 8 + 8 = 32
 Total visible faces = 56
But we have 44 Red cube
 minimum of number of white faces
cubes which are visible = 56 – 44 = 12
18. Multiple of 3  3, 6, 9, 12, 15, 18, 21, 24, 27, 30, 36 then n = 5, 8, 11, 14, 17, 20, 23......
Multiple of 5  5, 10, 15, 20, 25, 30, 35, 40, 45, 50, 55 then n = 8, 13, 18, 23, 28 ......
Now, common two digit numbers are 23, 38, 52, 67, 82, 97.

19. if start with H1  2 ways


if start with H2  2 ways
if start with W 1  2 ways

if start with W 2  2 ways


m=8
in case of circle
H1 H1

W2 H2 H2 W2
Hence n = 2

W1 W1
20. Total students = 300
One student read = 5 newspapers
Number of newspapers read by 300 students
= 5 × 300
= 1500 newspaper
1500
Number of different newspapers = = 25.
60

T PHYSICS

21. mg–T = ma a
T = m(g–a)
mg
4
= 6000 (10–2) = 4.8 × 10 N

u C v
22. A v B
5 5
Between A and B
v2 = u2 = +2a(2s) .....(i)
Between A and C
v’2 = u2 + 2as .....(ii)

u2  v 2
from (i) and (ii) v’ =
2

238 206
24. 92 U 82 Pb  824 He  601e

47
PAGE # 4747
CHEMISTRY
n
 1
31. Using the formula, N = N0  
2
where, No = initial amount of radioactive substance
N = Amount of substance left after ‘n’ half lives

Total time (t)


No. of half lives (n) =
Half life period

21.2 years
n=
5.3 years
n=4
4
 1 20
so, N = 20   = = 1.25g
2
  16

32. Zn(s) + H2SO4(aq)  ZnSO4(aq) + H2(g)


Zinc Sulphuric Zinc sulphate Hydrogen gas
acid (salt)

Zn(s) + 2NaOH (aq)  Na2ZnO2 (aq) + H2(g)


Zinc Sodium Sodium Hydrogen gas
hydroxide zincate

33. Acetylene (CH  CH) has a triple bond.

34. Ne has 10 electrons and oxygen has 8 electrons.

35. KMnO4 being strong oxidising agent will oxidise Cl– ion present in HCl to form chlorine gas.
2KMnO4 + 16 HCl 2KCl + 2MnCl2 + 8H2O + 5Cl2

36. Order of strength for halogen acids is HI > HBr > HCl > HF

38. Concentrated sulphuric acid is a dehydrating agent and it burns the organic compounds like sucrose so
the colour of the solution turns black.
Conc .H2SO 4
C12H22O11(s)    12C + 11H2O(g)
Sucrose Carbon Water vapour
(Black)

Oxidation

39. ZnO+C Zn + CO
Reduction

40. Monomer of Teflon is Tetrafluoroethene(CF2=CF2)

48
PAGE # 4848
PART-II (2 Mark)
MATHEMATICS
51. x = x3 + y4 ; y = 2xy
y = 2xy
Two cases 1. case -1 y=0&xR
1
2. case - 2 x= &yR
2
Now, equation x = x3 + y4
Taking case - I
x = x3
Solution x = 0, 1, –1
 pairs = (0, 0), (1, 0), (– 1, 0)
Taking case 2
3
1  1
=   + y4
2 2

3
= y4
8
1/ 4
3
y = ± 
8
 1  3 1 / 4   1  3 1 / 4 
   ,  
 Solution set =  2 ,  8   , 2 8 
     
52. f(x)  degree (n)
f(1) = 2
f(3) = 
f(x) = axn + bxn – 1 + cxn – 2 .......
If degree 0 f(x) = a = constant
f(1) = f(3) but it is not true
If degree 1 f(x) = ax + b
f(1) = a + b =
2
f(3) = 3a + b = 
Two variables & two unknown
a & b can be found uniquly
 one polynomial used only
For n>1
Let n = 2 ax2 + bx + c
We have 3 variable & only 2 equations can be formed from given condition
Hence infinite such polynomial can be formed

53. p(x) = Q(x) . (x2 – 3x + 2) + (2x – 3)


p(1) = – 1
p(2) = 1
at least one root between 1 & 2

–1 1 2
–1

49
PAGE # 4949
D x N x C
y

2y M
54. y

B
A 2x

y 2y
In MCN, tan  = and In ADN, tan (90 – ) =
x x
2y
cot  =
x
x
tan  = 2 y

2y2 = x2
x 2
y = 1

B ( 2 , 3)
S x R

x x

55. O(0, 0) P x Q (6, 0)


a A
2

3 x 3–x x 3
tan  = = = ....(i) and tan  = = ....(ii)
2 a 2 –a 6–a–x 6– 2

3a
x= [from (i)]
2

2x
 a=
3
and 6x – 2 x = 18 – 3a – 3x [from (ii)]

2x
 9x – 2 x = 18 – 3 . 3
 x=2

Q' P

102º
56.
r A r B
R' O
Since R can be any point between A & R’ & hence its corresponding point Q will lie on the arc AQ’. Hence
PRA can not be uniquely determined.

50
PAGE # 5050
3x y 2x
A B C D
57.
5y

AB 3
Given =
CD 2

BC 1
=
AD 5
Let AB = 3x CD = 2x , BC = y, AD = 5y
3x + y + 2x = 5y
5x = 4y
To find AC : BD = (3x + y) : (y + 2x)

3x  y 3x  5x / 4
2x  y = 2x  5 x / 4

3  5 / 4 12  5 17
= =
2 5/4 85 13

58. Let r = radius of sphere


and R = radius of cone
and H = height of cone

4 3 1
r = R2H
3 3
4r3 = R2H ....(i)
Now,

k4r2 = R2 + R R 2  H2

k(4r2) = R2 + R R 2  H2

 6 
2
 R  R 2  16r 
4kr = R  R4 
 

 r2  6 


 =1 1  R 2  16r  r2
 4k  2   R6  Let =x
R    R2

 4kx = 1 + 1 16 x 3
 16k2x2 – 8kx + 1 = 1 + 16x3
 x=0 16k2x – 8k = 16x2
2x2 – 2k2x + k = 0 since cone is unique therefore it has equal roots
4k4 – 8k = 0
k = 0, (2)1/3

51
PAGE # 5151
59. Let original four digit number is = m2
and new 4-digit number is = n2
then m2 – n2 = 1111 (where, m, n, )
(m + n)(m – n) = 1 × 1111 or 11 × 101
Case - I
m + n = 1111
m–n=1
m = 556
n = 555
But m2 = is a 4-digit number
Case - II
m + n = 101
m – n = 11
m = 56 and n = 44
So, there is only one such 4-digit number.

60. m , n are integer


1/3 < m/n < 1 ...(i)
m m
= n0
n n
m
= = +ve integer
m 1
 m=2
Using equation (i)
n = 3, 4, 5
m 2 2 2
 = , ,
n 3 4 5

PHYSICS
1 2
61.  = 0t – t
2
1
 = 60 × 5 – × 8 × 25
2
= 300 – 100
 = 200 radian
 100
or n = 
2 
~
 32rev./s

62. Energy released = mLV


= volume of water × density ×LV
= 100 × (103)2 ×1×103 × 22.5 × 105 J
= 22.5 × 1016 J
22.5  1016
Number of bombs =  2250 ~
 2000
1014

5r 5  1 5
63. Req =   
6 6 6
 V  10 volt
V 10  6
So, I =   12 A
R 5

52
PAGE # 5252
64. time taken in rotataing angle is t, then N

45  10 3
t= 8
 15  10 5 s
3  10 27º

t = 15 ×10–5 second
360
T=  15  10 5

1 1 1000
N=  3
 = 250 rev/s
T 4  10 4

65. Focal length, f = 1m,


size of object, h1 = 1000m, size of image, h2 = 0.2 m
image is real, so
h2 v 4
– h  u  v  2  10 u
1

1 1 1
  
f v u
1 1
1=– 4
  u = – 5000 m or u = –5 km
2  10 u u

CHEMISTRY

66. N1V1 = N2V2


36 × V1 = 3 × V2
V2
= 12
V1

67. 4Fe + 3O2 + nH2O  2Fe2O3.nH2O


Iron Oxygen Water Hydrated Ferric oxide
(Rust)

68. Solubility of KNO3 at 90º C = 200 g/100 ml


Solubility of KNO3 at 30º C = 40 g/100 ml
Decrease in solubility when KNO3 solution is cooled from 90ºC to 30ºC = 200 – 40 = 160 g/100ml
Yeast KMnO 4
69. C6H12O6 – 
2CO
 2C H OH 
2 5
 CH3COOH
2

70. Zn + 2H2SO4(conc.)  ZnSO4 + SO2(g) + 2H2O


(CH3COO)2 Pb + SO2  PbS
(Black Precipitate)
Cu + SO2  CuS + O2
(Black
mirror)

53
PAGE # 5353
HINTS & SOLUTIONS (YEAR-2008)

ANSWER KEY
Ques. 1 2 3 4 5 6 7 8 9 10 11 12 13 14 15
Ans. A B D A C C C A C D B B D B A
Ques. 16 17 18 19 20 21 22 23 24 25 26 27 28 29 30
Ans. D D A A B C A D A B C B D C A
Ques. 31 32 33 34 35 36 37 38 39 40 41 42 43 44 45
Ans. B A A C D A C A D C A B B C D
Ques. 46 47 48 49 50 51 52 53 54 55 56 57 58 59 60
Ans. B B B B A C A&C C A D C A D B D
Ques. 61 62 63 64 65 66 67 68 69 70 71 72 73 74 75
Ans. B A D A C D A B A A C B D B A
Ques. 76 77 78 79 80
Ans. B C C A D

PART-I (1 Mark)
MATHEMATICS
1. Obvious (A) is greatest
1 2 3 n
2. S=  2  3  ...  n  .... 
10 10 10 10
S 1 2
= 2 + + ......... 
10 10 103
Subtracting,
9S 1 1 1
= + 2 + + ......... 
10 10 10 103

1
9S 10
= 1
10 1–
10

9S 1
=
10 9
10
S=
81

3. (1024)1024 = (16)16n
(210)1024 = (24)16n
10 × 1024 = 4 × 16n
10  1024
n=
4  16
n = 160

4. x2 + 6x + 8
xR
x2 – 2x – 8  0
x2 – 2x – 8 = x2 + 2x – 4x – 8
x(x + 2) – 4(x + 2)  0

54
PAGE # 5454
2
x + 6x + 8
–4 –3 –2 0 2 4

x  [–2, 4]
clearly min value of expression is 0
at x = – 2

5. Check by option
P12 = {24, 36, 60, 84, ....}
P20 = {40, 60, 100, ......}
P12  P20 has common element

6. All even values of a i.e. 50 and 1, 9, 25, 49, 81, total 55

7. If any statement is true then remaining 2 are false.

D
P
4 2 4 2
8. F
4 2
B
A E

Angle bisector  Incircle is formed whose radius = 4 2


PE = r = 4 2
PF = r = 4 2 also PF = AE
 APE, (AP)2 = (AE)2 + (PE)2
= ( 4 2 )2 + ( 4 2 )2 = 64
 AP = 8

1 D
9. Area of rhombus = dd C
2 1 2
Let one diagonal = x y y h
1 y
= ×(x)(2x) = x2 A F
2 2 2 E B y2 – h 2
y –h
A = x2
Let side of rhombus = y & height = h

BFC side BF = y 2  h2

InAFC, (y + y 2  h 2 )2 + h2 = (AC)2 = 4x2

DEB (y – y 2  h 2 )2 + h2 = (BD)2 = x2
Adding
4y2 = 5x2

5x 2 5A
y= =
4 2

55
PAGE # 5555
A(2a)

(a) E
P
10.
B C (3b)
D
(2b)
 
Let B is origin and the position vector of A and C are 2a and 3b
 
Then P.V. of E = a and P.V. of D = 2b
Now, let P divides AD in  : 1 ratio
and P divides EC in  : 1
   
2b  2a 3b  a
 =
 1  1
       
2b  + 2b  + 2a  + 2a = 3b  + a  + 3b + a
 
a (2 + 2 –  – 1) = b (3 + 3 – 2 – 2)
 
But a and b are not collinear..
2 –  + 1 = 0 and  + 3 – 2 = 0
We get  = 1
 
a  3b
Now, P.V. of P is =
2
   
1   a  3b    a  3b 
a     2b  
ar PED 2 
 2   2 
Now, ar ABC = 1  
2a  3b
2

1    
4
 
a  3b  b  a
1

=   =
6ab 12

N
D C

O M
11. y P

A B
L

Let AB = a, BC = b
and PL = h1
then PN = b – h1
and OP = h2, then PM = a – h2
1
 ar(PAB + PCD) = a(h1 + b – h1)
2
ab
=
2
1 ab
and area (PBC + PAD) = × b × (h2 + a – h2) =
2 2
From this only option B is correct

56
PAGE # 5656
12. Let x, x + 1, x + 2, x + 3, x + 4, x + 5, x + 6
& x + 7, x + 8, x + 9, x + 10, x + 11
7x + 21 = 5x + 45
2x = 24
x = 12
largest = x + 11 = 23

1
13. Let x minite will be taken. In one minute A can fill the part of tanker and in one minute B can fill the
60

1
part.
40

Both can fill in t

t t
+ =1
60 40

60  40
t=
100
t = 24
1
both can fill in one minute part of tanker..
24

x 1 x 1 
1=   +  
 2  40 2  24 

x x
1= +
80 48

80  48
x= = 30
128

14. Given a < b < c


 6 three digit number are possible with distinct a, b, c.

a b c
 a c b
 b a c
 b c a
 c a b
 c b a
1 5 5 4
= 100[2(a + b + c)] + 10[2(a + b + c)] + [2(a + b + c)] = 1554
111[2(a + b + c)] = 1554
 a+b+c=7
Given a < b < c
 1, 2, 4 only satisly above two condition.
Hence, c = 4.

15. Since factor of 128 are = 1, 2, 4, 8 , 16, 32, 64, 128


Hence it will be increased by 8.

57
PAGE # 5757
PHYSICS
16. speed will not decrease, so answer is (D)

2s
17. For electron, t1 = ae

2s
For protion, t2 = ap

t2 ae eE mp
or t  ap
 
me eE
1

mp
=
me

18. Focal length, f = 6 cm


u = 1.5m = 150 cm
v=?
1 1 1
 
f v u
1 1 1
 
6 v 150
1 1 1 25  1
  
v 6 150 150
150 75 25
v=    6.25
24 12 4
change in distance = 6.25 – 6 = 0.25 cm
= 0.25cm = 2.5 mm decreased

19. Initial momentum, P1 = mvcos30


and final momentum, P2 = mvcos30
change in momentum
P = – 2mv cos30
P = – 3 mv
Force on wall-1
2mv
F1 =
t
Force on wall-2
3mv
F2 = , so F1 > F2
t

22. A1u1 = A2u2


u1 A 2 1
 
u2 A1 16

23. resultant force at centre is zero. On removing the charge from the position 6, the resultant force at centre
kq
will be downward.
r2

58
PAGE # 5858
v d
25.  w
V dL

1 dw
  dw = 420 kg/m3
2 840
420
R.D. =  0.42
10 3

26. RS = nR for maximum resistance


RP = R/n for minimum resistance
RS
or R = n2
P

27. For block 2kg


T
T – 2g = 2a ........(i) T
For 6 kg
a a
6g – T = 6a ........(ii)
2g
From (i) and (ii) T = 30 N
6g

28.  v e  2gR with height g will change, so answer is (D)

partial pressure
29. R.H. =
saturated vapour pressure

90 partial pressure
=
100 0.0169  10 5
partial pressue = 0.0152 × 105 Pa

CHEMISTRY
31. NaOH HCl
N1V1 = N2V2
0.5 × V = 2 × 10
V = 40 mL

35. Ethanol (C2H5OH) and dimethyl ether (CH3 –O – CH3) have same molecular formula but different functional
groups, so they are isomers.

36. For the elements belonging to one period, increase in atomic number results in decrease in atomic
radius. So Li has the largest atomic radius.

37. 2H2O2  2H2O + O2

39. S + O2  SO2


1 mole 1 mole 1 mole
1 1 1
mole mole mole
2 2 2
3.01 × 1023 0.5 mole ?
23
 3.01 × 10 molecules of SO2 will be formed.

59
PAGE # 5959
40. Zn and Pb are placed above hydrogen in the metal activity series, so they will produce hydrogen gas with
dilute acids.

41. Milk of magnesia is basic, water is neutral and lemon juice is acidic in nature.

42. As pressure is increased, solubility of gas in liquid increases.

Conc. H2SO4
45. CH3 – CH –CH2 – CH2 – CH3 CH3 – CH = CH – CH2 – CH3 + CH2 = CH – CH2 – CH2 – CH3
–H 2O
2-Pentene (Major) 1-Pentene (minor)
OH
2-Pentanol

PART-II (2 Mark)
MATHEMATICS

61. x 1        
 2 x1  1 + x 2  4 x 2  4 + x 3  6 x 3  9 + x 4  8 x 4  16 + x 5  8 x 5  25 = 0 
2 2 2 2 2
  x  1  1 +  x  4  2
1 2
+  x  9  3 + 
3 x4  16  4  +  x 5  25  5 
Now, x1  1 – 1 = 0, x 2  4 – 2 = 0, x 3  9 – 3 = 0, x 4  16 – 4, x 5  25 – 5 = 0
x1 = 2, x2 - 8, x3 = 18, x4 = 32, x5 = 50
x1  x 2  x 3  x 4  x 5
 = 55
2

62. (1 + 2x + 3x2 + .... 21x20) (21x20 + 20x19 + 19x18 + ........ 2x + 1)


coeff. of x30 is 11.21 + 12.20 + ..................... + 21.11
= 2[11.21 + 12.20 + 13.19 + 14.18 + 15.17] + 16.16
= 2[231 + 240 + 247 + 252 + 255] + 256
= 2[1225] + 256
= 2450 + 256
= 2706

63. If we follow the pattern according the rule that angle made by incident ray with normal is equal to the angle
made by reflected ray with normal then we find that ball will not go in any hole.

64. C can be collinear with A & B.

65. Let initial prize is P


 Px 
after X% increment  P  
 100 
after decrement y%
 Px   Px  y 
 P   – P    =P
 100   100  100 

x y xy
1+ – – =1
100 100 (100 )2

x–y xy
=
100 (100 )2

1 1 1
y – =
x 100

60
PAGE # 6060
PHYSICS
66. mass of water, m1 = 0.4 kg
temperature of water, 1 0ºC
mass of ice, m2 = 0.1 kg
tempeature of ice, 2 = – 15ºC
mass of steam = m kg
final temperature of mixture  = 40ºC
specific heat of ice, sice = 2.2 × 103 J/kg×k
latent heat of fusion, Lf = 333 × 103 J/kg
latent heat of vaporisation, LV = 2260 × 103 J /kg
Heat given = Heat taken
By steam of 100ºC to water of 100ºC + By water from100ºC to 40ºC = By water from 0ºC to 40ºC + By ice
from –15ºC to ice of 0ºC + By ice of 0ºC to water to 0ºC + By water of 0ºC to 40ºC
mLv + mSw (1100 – 40) = m1 × Sw (40 – 0) + m2sice(15) + m2 × Lf + m2Sw (40–0)
m (2260 × 103 + 4200 × 60) = 0.4 ×4200 × 40 +0.1 ×2.2 × 103 ×15 + 0.1 × 333 × 103 + 0.1 × 4200 × 40
m (2512 × 103) = 67200 + 3300 + 33300 + 16800
120600 603
m=  kg = 48 g
251200 12560

100

67.

u v
As for question
u + v = 100 cm .....(i)
after displacing lens by 40 cm u and v will be
u + 40, v - 40
for Ist condition
1 1 1 1 1
(i)    
f v ( u) v u

1 u  v 100
 
f uv uv
(ii) For second condition
1 1 1
 
f v  40  (u  40)

u  40  v  40 uv
= =
( v  40)(u  40) ( v  40)(u  40)
From (i) and (ii)
100 100

uv ( v  40)(u  40)
v– u = 40 .....(ii)
v + u = 100
from equation (i) and (ii)
2v = 140
v =70, u = 30
1 uv 100
 = f = 21 cm
f uv 2100

61
PAGE # 6161
68. we know that
A1V1 = A2V2 = Q (volume flowing per second)
25 × 0.6 = 12 × v2
v2 = 15 m/s ........(i)
Q = A1v1=(5 ×10– 2)2 × 0.6 = 4.71g
force, F = rate of change of momentum
F = mv2 – mv1
= 4.71 × 15 – 4.71 × 0.6 (m = 4.71, mass flow per unit time)
F = 67.9 N
CHEMISTRY
71. Consider the volume of the solution = x cm3
Then the mass of the solution will be = 1.13x
(mass = density × volume)
The solution contains 18% of NaCl by weight
18
 × 1.13x = 36
100
3600
x= = 177 cm3
18  1.13
+
H
72. CH3 – CH2 – COOH + CH2 – OH CH3 – CH2 – COOCH2CH3 + H2O
Propanoic Ethyl propanoate
acid CH3
Ethanol

73. Consider that the salt contains x molecules of water .


Molecular weight of anhydrous salt = 160 g
so molecular weight of hydrated salt will be = 160 + 18x g
10
Then, no. of moles of water present in 10x gm of hydrated salt = ×x
160  18 x
10 x
and weight of water present in 10 gm of hydrated salt = × 18
160  18 x

Hydrated salt  Anhydrous salt + Water


10g 6.4 g 3.6g
180 x
= 3.6
160  18 x
180x = 576 + 64.8 x
x=5

74. Cr2 O72 – + 6Fe2+ + 14H+  6Fe3+ + 2Cr3+ + 7H2O

Change in oxidation number of Cr is = 6 – 3 = 3


Change in oxidation number of Fe is = 3 – 2 = 1

75. CaCO3 + H2O + CO2  Ca(HCO3)2



Ca(HCO3)2  CaCO3 + H2O + CO2

62
PAGE # 6262
HINTS & SOLUTIONS (YEAR-2009)
ANSWER KEY
Ques. 1 2 3 4 5 6 7 8 9 10 11 12 13 14 15
Ans. D A A A C C B D D B D B D B C
Ques. 16 17 18 19 20 21 22 23 24 25 26 27 28 29 30
Ans. C B C B A A D A D C D C B D B
Ques. 31 32 33 34 35 36 37 38 39 40 41 42 43 44 45
Ans. D C A D B C A C D B C B A B C
Ques. 46 47 48 49 50 51 52 53 54 55 56 57 58 59 60
Ans. D C C B A B A A D B B A A C C
Ques. 61 62 63 64 65 66 67 68 69 70 71 72 73 74 75
Ans. A B B C D C A D D A A C D B C
Ques. 76 77 78 79 80
Ans. C D A A B

PART-I (1 Mark)
MATHEMATICS
1. x5 0  1– x  0
x<1 ...(i)
x5 > 1 – x
x + 5 > 1 + x2 – 2x
x2 – 3x – 4 < 0
(x – 4) (x +1) < 0
x  (–1, 4) ....(ii)
Using (i) & (ii) x  (–1, 1)

3. x = x2 + y2 & y = 2 xy
 when y = 0
 x = x2
 x = 0, x = 1
 solutions are (0,0) (1,0)
1
 when x =
2
1 1 1 1
 – = y2  = y2  y=±
2 4 4 2
 1 1  1 1
 solutions  ,   ,– 
2 2 2 2
4. x3 – 3 |x| + 2 = 0 [Let x > 0]
x3 – 3x + 2 = 0
(x – 1) (x2 + x – 2) = 0
– 1 9
x = 1, x = =1
2
Now, let x < 0
x3 + 3x + 2 = 0
no solution
x = 1 only one solution

PAGE # 63
5. (1 + 2x)20 = a0 + a1x + a2x2 + ------- + a20x20
put x = 1 320 = a0 + a1 + a2 + ------- + a20
x = –1 1 = a0 – a1 + a2 – a3 ------- + a20
subtract 320– 1 = 2(a1 + a3 + -------- + a19]
add 320 + 1 = 2 (a0 + a2 + -------- +a20]
3 20
2 (a1 + a3 + ------- + a19) + 3 [a0 + a2 + --------- + a20] = (320–1) + (3 + 1)
2

5.320  1
=
2
6. Points which are equidistant from O and P1 lies on perpendicular bisector of OP1

P5
P1
P4
O
P3 P2

Similarly for others


 In figure points on pentagon are equdistant from points P1, P2, ...... and O
 interior region of pentagon is closed to ‘O’

7. Let (x, y) be any point

Using given condition 2 ( x  2)2  y 2 < 3 ( x  0)2  ( y  3)2


5x2 + 5y2 – 54y + 16x + 65 > 0
x2 + y2– 10.8y + 3.2x + 13 > 0
It is a circle, radius = 18.72
Centre (– 1.6, 5.4)
Hence region is exterior of (x + 1.6)2 + (y – 5.4)2 = 18.72.

C
8. (r + 3)2 + (r + 10)2 = (13)2 3
 r2 + 9 + 6r + r2 + 100 + 20r = 169 3 D
 2r2 + 26r – 60 = 0
10
 r2 + 13r – 30 = 0 r
 (r + 15) (r –2) = 0 A
B r 10
 r=2

9. To find possible intergal value of 4th sier minimum possible value of 4th side greater than 0 is 1.
For maximum possible value.
D

20
C
A
B
let angle ,  are slightly smaller than 180º
if ,  = 180º
AD = 35
 maximum value of 4th side is 34
 1, 2, 3, ------------- 34 are possible value of 4th side
 34 values are possible.

PAGE # 64
4
R' 3
V' 172 .8 3
10. = =
V 100 4
R 3
3

R'
= 1.2
R

S' 4 R' 2
Now, ratio of surface area = = =
S 4 R 3

S'
= = 1.44
S

Hence surface area increased by 44%

11. x = 0.d25 d25d25 ------

x = 0. d25

1000 x = d25. d25


999x = d25

d25
x=
999

d25
x=
37.27

25
take d = 9 then x =
27
d = 9 n = 25
d + n = 34

12.  Let at x minute past 10 o’clock they become symmetric


We know the speed of hour hand and minute hand is 1 : 12

x
 When minute hand moves x mimute distance then hour hand moves minute distance
12
x x1
12
x 11
 = 10 – x
12
10 – x 2
10
13 x
 = 10
12

120
 x=
13

 x = 9 minute 13.8 second therefore required time is 10h 9m 13.8 sec.

PAGE # 65
13. P = LLLLLL W
9 8 7 6 5 4 1
P=  ×  ×  ×
10 9 8 7 6 5 4
1
P=
10

14. let x student like all three games.


 If x minimum then a + b + c will be maximum p q
and p + q + r should be minimum. Cricket a Football
 p+q+r=0 x
b c
a + b + x = 74 ..... (i)
a + c + x = 76 ......(ii)
b + c + x = 82 ......(iii) r
and a + b + c + x = 100 Tennis
a + b + c = 100 – x
Add (i), (ii) & (iii)
2 (a + b + c) + 3x = 232
2 (100 – x) + 3x = 232
200 – 2x + 3x = 232
x = 32.

15. Number of integers between 2n and 2n + 1 is 2n + 1 – 2n – 1


and term = 2n + 1
lastterm = 2n+1 – 1
n 1
2  2n  1 n
 Sn = [2 + 1 + 2n + 1 – 1]
2
n 1
2  2n  1 n 
= (2 )(1 + 2)
2
n
= (2n – 1)
2 .3
2
Sn = 9 ;  
 3 × 2n – 1 × (2n – 1) = 9
2n – 1 × (2n – 1) = 3
2n(2n – 1) = 6
It is possble when n is even.
PHYSICS
17. given mass of planet A = mA
and mass of planet B = mB
mB = 8 mA
Gm A GmB
gA = r and gB = r
A2 B2

2
gA mA  rB 
or    .......(i)
gB mB  rA 
 mB = 8 mA

4 3 4  rB
rB d  8  rA3 d   r =2
3 3  A

gA 1 1
 4 
gB 8 2
gB = 2gA

PAGE # 66
25. A = r2
A A
2
A r
A  A 
%  2   100
A  r 

A
%  2  0.15 = 0.30%
A

235 207
27. 92 U 82 Pb  724 He  4 1e0
so, n = 4 ( –1 particles)

29 . Bulbs are connected in series and resistance of 100W is greater then that of 200 watt.
In series, P = I2 R (Here I is constant)
So, PR
So, power of 100W will be greater in the combination.

30. As masses are equal for cude and sphere


So, ms = vs × density
me = ve × density
as ms = me vs × density  ve × density
vs = ve
4 3
 r  s3
3
surface area comparison 6s2 > 4r2

CHEMISTRY

31. Silicon is tetravalent, so it forms SiCl4 .

32. NH4Cl undergoes sublimation while NaCl does not.

Oxidation

34. SO2 + 2H2S 3S + 2H2O


Reduction

36. As per Boyle’s law


PV = constant
1
and P 
V

38. CaC2 + 2H2O  Ca(OH)2 + C2H2


Calcium Water Calcium Ethyne
carbide hydroxide (acetylene)

39. As we move downwards in a group, atomic radii increases. So the order will be Li < Na < K < Cs.

40. CH3 – C  CH (Propyne) and (cyclo propene) are two possible structural isomers of C3H4 .

PAGE # 67
41. Propionic acid and methyl acetate both have same molecular formula (C3H6O2) but different functional
groups, so they are isomers.

42 N2 + 3H2 2NH3
Initial
no. of
moles 1 0.5 0
After
reaction 1 – 0.167 0.5 – 0.5 0.334 mole

0.334 moles of NH3 = 2.0 × 1023 molecules

43. CH2OCOR CH2 – OH


Alkaline
CHOCOR + 3NaOH CH – OH + 3RCOONa
Hydrolysis
Soap
CH2OCOR CH2 – OH
Triester Glycerol
44. Copper cannot displace lead from its solution as it is less reactive than lead..

PART-II (2 Mark)
MATHEMATICS
61. loga b = 4 & logcd = 2
b = a4 d =c2
a4 – c2 = 7
(a2 – c) (a2 + c) = 7
a2 – c = 7 & a2 + c =1 not possible
or a2 – c = 1 & a2 + c = 7
2a2 = 8
a = ±2
a=2 c=3
 c–a=1

62. P(x) = 1 + x + x2 + x3 + x4 + x5
1 x6
=
1 x
It has 5 roots let 1, 2, 3, 4, 5 they are 6th roots of unity except unity
Now, P(x12) =1 + x12 + x24 + x36 + x48 + x60 = P(x)  Q(x) + R(x)
Here R(x) is a polynomial of maximum degree 4
Put x = 1, 2, .......5
we get
R(1) = 6, R(2) = 6, R(3) = 6, R(4) = 6, R(5) = 6
i.e. R(x) – 6 = 0 has 6 roots
Which contradict that R(x) is maximum of degree 4
So, it is an identity
 R(x) = 6
63. BD  b and CE  c
c sin A  b ...(i) A
and b sin A  c ...(ii)
c D
(i) + (ii) E
b
(c + b) sin A  b + c k
sin A  1 h
B C
 A = 90º a
Now from (i) and (ii)
c  b and b  c
 c=b Hence angles are 45º, 45º, 90º

PAGE # 68
A
(37 )2  (7)2 – (33 )2 (37 )2  x 2 – (33 )2
64. cos  = =
2.37.7 2.37.x
37 33 37
[(37)2 + (7)2 – (33)2] x = 7(31)2 + 7x2 – 7(33)2
x = 40
B 7 D x C

65. Case - I

a r

A1 =

Case-II

a/2 a/2

r – a/4
Case-III
30º

a/2
r
a/4
a/2

a/2
r + a/4

 a a
A3 = a  r   r   = 2ra
 4 4
Hence A1 = A2 = A3

PAGE # 69
PHYSICS

C
h=? u=0

67.
d

Time taken by the photon, t = d/c

1 2
h=0×t+ gt
2

1 d2
h= g
2 c2

gd2
h=
2c 2

69. For lens L1,


u= –0.40 m
f = 0.20 m

1 1 1
 
v f u

1 1  1 
  
v 0.20  0.40 

1 2 1
  v = 0.40 m
v 0.40
 for lens L2 , u = 0.1 m
f = – 0.1 m

1 1 1
 
v 0.1 0.1
v= 

70. After removing charge from P, net force on central charge will be :

Kq1q2 9  10 9  10 5  5  10 5
F= 2

r 12

F = 4.5 N
m = 0.5 kg
so, acceleration,

F 4.5
a=  = 9 m/s2 upwards
M 0.5

PAGE # 70
CHEMISTRY
71. 2NaCl + H2SO4  Na2SO4 + 2HCl(g)
(X)
2HCl + CaCO3  CaCl2 + H2O + CO2
(X) (Y)

Ca(OH)2 + CO2  CaCO3 + H2O


lime water (Y) Milky
Suspension

72. Initially concentration of salt in solution

222  10 –3
= = 0.2M
111  10  10 – 3

On dilution the final concentration of CaCl2 will be


M1V1 = M2V2
0.2 × 10 = M2 × 100
M2 = 0.02 M

CaCl2  Ca2+ + 2Cl–


0.02 M 0.02 M 2 × 0.02
[Cl ] = 0.04M = 0.04 mole/L

73. 4Al + 3MnO2  3Mn + 2Al2O3


To reduce 3 moles of MnO2 required moles of Al = 4
So, for one mole of MnO2 required moles of Al will be = 4/3

Alkaline
74. CH3 – CH2 – OH   CH3COOH
KMnO 4
Ethanol Acetic acid
(X)

H
CH3COOH + CH3OH   CH3COOCH3 + H2O
Acetic acid Methanol Methyl acetate
(Y)

75. On dilution, mili equivalent of the solute remains constant.


Initially pH of HCl = 4
so normality of HCl = 10–4 N
after dilution pH of HCl = 5
so normality of HCl will be = 10–5 N
N1V1 = N2V2
10–4 × 10 = 10–5 × V
V = 100 mL
So, 90 mL of water should be added for this pH change

PAGE # 71
HINTS & SOLUTIONS (YEAR-2010)
ANSWER KEY

Ques. 1 2 3 4 5 6 7 8 9 10 11 12 13 14 15
Ans. C D B A B B A D D D B A C D C
Ques. 16 17 18 19 20 21 22 23 24 25 26 27 28 29 30
Ans. D A D B A D A A B B A A D C C
Ques. 31 32 33 34 35 36 37 38 39 40
Ans. D C A D D A C B C B

PART-A (1 Mark)

MATHEMATICS

1. x2 + bx + a = 0

x2 + ax + b = 0

x=–1  x+1=
(x + 1)2 + a(x + 1) + b = 0
x2 + (a + 2)x + 1 + a + b = 0
x2 + bx + a = 0 and 1+a+b=a  b=–1
a=–3
_________
a+b=–4

t
2. 3x/y = t 3t – = 24  8t = 3 × 24
3
 t=9
So, 3x/y = t  3x/y = 9
 3x/y = 32  x = 2y.

xy 3y
  = 3.
x–y y

n(n  1)(2n  1) 2n  1 3k – 1
3. 2  =k  n=
6n(n  1) 3 2

3k – 1 20 1
1   100  3  3k 201  1 k 
2 3
1  k  67.
 Number of odd integers = 34.

PAGE # 7272
4. 2 + b2 = 392, b2 + h2 = 402, h2 + 2 = 412
Adding
2(2 + b2 + h2) = 392 + 402 + 412

392  402  412 ( 40 – 1)2  402  ( 40  1)2


 2  b2  c 2 = =
2 2

3( 40)2  2 4802
= = = 2401
2 2
= 49.

5. It has to be an isosceles triangle.

1 2 1 1
= ×1 x – = 4x 2 – 1
2 4 4
Perimeter = 1 + 2x  odd which is always irrational.

1
6. = × 12 × 6
2
= 36

7.

22 2 2
60  a  60  3a 
A1 = A5 =    = a , A = A =    = 3a 
360  2  24 2 4 360  2  8

2 2
240  5a 
A3 =    = 25a 
360  2  6

 a2   3a 2   25a 2 
Area that can be grazed = 2 24  + 2 8  +
   
= 5a2.
    6

PAGE # 7373
dv
8. = c.
dt
1 2 r
v= r h tan  =
3 h
 3
= h tan 
3
dv dh h3 tan 
= h2 tan   ct + k =
dt dt 3
H3 tan   3
when t = 0, h = H  k=  ct = (h – H3) tan 
3 3

H  –7
when t = 21 , h = c= 3 H3  
2 6  8 
h=0
  3 – 7 

(–H3 ) tan  =  3 H    t  7 = 7
t t = 24
3 6  8  24 8  72
More time in minutes does it empty the vessel is 3

9. Water + solid = 1000


99
Water is  1000 = 990
100
water evaporated is x.
 990 – x 
so  100 = 98
 1000 – x 
99000 – 100 x = 96000 – 98x
1000 = 2x x = 500
10.

 2x 2  30
xy = 10 and yz = 4  z= y  5 also  x  a = 12  a=
5  x

 30  x x 50
and   (b) = 15  b= and   (c) = 25  c=
 x  2 2 x

 50 
 Area = x   = 50
 x 

PAGE # 7474
PHYSICS


11. T = 2 g

First distance of com from suspension point will increase then decrease. So,
 T.

12. when sliding has started (KVPY / 2010 / SA)

till acceleration of block is zero F – fk = ma


F – fs = 0
fs = F

 49  500
13. Mg = 40 ×   ×
 1000  1

40  49  5  10
M= = 100 kg.
10  98

14.

GMm mv 2
=
r2 r
No change because distance between them will be from centre to centre distance which is unchanged.

PAGE # 7575
15.

Since, ig = 0
PR = QS
Still it will be a balanced W.S.B.
So, again ig = 0.

KQq 1 Qq
16. 2 = 4 . 2
R 0 R

Since initially net force on Q was zero by symmetry


 
So, F1  FRe maining 11  0
 
FRe maining 11  F1

1 Qq
So, 4 . 2 towards the position of the removed charge.
0 r

Vi2
17. Pi =
Ri

Since, Rf < Ri keeping V = constant


V = Vf

V2
Pf =
Rf

Since, R  P  .

18.

the combination will behave as parallel slab so light get laterally displaced without any spectrum.

19. 20ºC 80ºC


S  as T 
d = m.s.d d = ms d
Since, average S of body which is initially at 80ºC is higher then body initially at temperature 20ºC so
temperature decreases of earlier will be less then temperature increases of letter.
So, Tf > 50ºC.

PAGE # 7676
20. tx = 3 T + 300 Q = ms 
tx = 3T

Q Q
S= S=
m. m.
Since, unit of  is joule in both system
X T
m = m0kg m0kg
Q = Q 0J Q0.J
tx T

Q0 Q0 3Q
Sx = m t = 1400 ST = m T = m t
0 x 0 0 x

ST = 3 × 1400 = 4200 J-kg–1K–1


CHEMISTRY

21. Aqueous solution containing more number of particles have more elevation in boiling point.

22. 14
Si : 1s2 2s2 2p6 3s2 3p2


23. CaCO3 (s)  CaO (s) + CO2 (g)

 25   25 
Number of mole    
 100   100 

 25 
Amount of CO2 =   × 44 = 11 gram.
 100 

24. As we move ‘left to right’ in 2nd period, atomic radii decreases due to increase in effective nuclear charge.

25. In BCl3 octet rule is not satisfy.

Total number of 6 electrons in outermost shell of B after bonding.

26. MnO2 + 4HCl  MnCl2 + 2H2O + Cl2


Cl2 gas produces.

27. C4H7Br
CH2 = CH – CH2 – CH2 – Br

H H H H
| | | |
H – C  C – C – C – Br
| |
H H

Number of covalent bond = 12.

PAGE # 7777
28. pH = 2  [H+]i = 10–2 M
pH = 5  [H+]f = 10–5 M

[H ] f 10 5  1 
= =  

[H ]i 10 2  1000 

So, H+ concentration decreases thousand fold.

29. For 1st jar :

2
Number of moles of H2 (g) = =1 mole.
2
Number of molecules of H2 (g) = 6.02 × 1023.
For 2nd jar :

28
Number of moles of N2 (g) = =1 mole.
28
Number of molecules of N2 (g) = 6.02 × 1023.
So, both jar have same number of molecules.

30. and

(cis) (trans)

cis and trans are stereoisomeric pair.

PAGE # 7878
DESCRIPTIVE TYPE QUESTIONS
PART-B (5 Mark)

MATHEMATICS

1. Let total amount is n2


Total borrowed amount = (2u + 1) 10
n2 – (2u + 1) 10 < 10
True for. n = 6 u=1
So, the left amount = 6.

2.

Let  ADE is equilateral and D is mid point of AB and E is mid point of AC


[given condition is true for above assumption]
 Area of quad. ADPE = Area of quad. DPFB
= Area of quad. EPFC
 Area of  ABC = 12 sq. units

3. From the question if m = 1111 or


(i) m = 111.11
is always divisible by n = 11 which is coprime with 10
(ii) by choosing a = k 10b (10c – 1) when k is any natural number we can option any natural number k.
The problem seen to have an error which may be due to memory retersion constraints.

PAGE # 7979
PHYSICS

4.

Assume to be spherical concave.

(R  H)
cos0 =
R

(i) P.E. = mg(R – R cos) = mgR(1 – cos)


(ii) mgH – mgR (1 – cos) = kinetic energy

d2 
(iii) m(g sin)R = (mR2)
dt 2

2
   
1 0 
2
T g  16      
tPQ =  2

= 2 1 0 
4 4 g  16 

(iv)

mV 2
N – mg =
R

mV 2
N = mg + where V = 2gh .
R

PAGE # 8080
5.

(i) For RP

 RR V  RR V  RR A  R AR V
R estimated    R A  RP  R 
 R  R V  R  RV

V RR V
RP = =
 R  R V + RA

 R.R V 
(ii) RP = R     R A
 R  RV 
Since, RA < < R < < RV
RR V
Rest =  RA ~
– R
R  RV
 RP = 0

(R  R A )R V
(iii) RQ V = 
RA  R  RV

(R  R A )R V
Restimated =
RA  R  RV

(R  R A )R V
RQ = R –
R  RA  RV

R 2  RR A  RR V  RR V  R A R V
=
R  RA  RV

R 2  RR A  R A R V
=
R  RA  RV

R 2 RR A
=   RA
RV RV
RQ ~
– RA .

R 2  RR A  R A R V
(iii) =0
R  RV  RV

After solving, R= R AR V .

PAGE # 8181
6.

1 1 1 1 1 1 2  1
(i)     =
V  20  10 V 10 20 20

20
V= 
3
(ii) fLM = –5 cm
20
+ d = 10
3

20 10
d = 10 – = cm
3 3

(iii)

(A) Ist reference with lens


20
V= 
3
(B) Then mirror,
Xim = –X0 M
(C) Again by lens,
1 3 1
 =
V  40  10

1 1 3 4  3
  =
V 10 40 40

40
V= cm
7
40
It means right of lens at a distance cm.
7

CHEMISTRY

7. (I) (A) Bottle-3 does not react with HCl or NaOH.

(B) Bottle-2 reacts only with NaOH.


(C) Bottle-4 reacts with both NaOH or HCl.
(D) Bottle-1 reacts with HCl only.
(II) Bottle-4 is highly soluble in distilled water due to zwitter ion formation.

8. (i) Balanced equation are :


(a) 3 Cu + 8 HNO3  3 Cu(NO3)2 + 2NO + 4H2O
(b) 2 Cu2  Cu22 + 2
(c) 2 Na2S2O3 + 2  Na2S4O6 + 2 Na

PAGE # 8282
2.54
(ii) Mole of 2 = = 0.01
254
Mole of Cu2 = 2 × mole of 2 = 0.02
Mole of Cu2 = mole of Cu = 0.02
wt. = 0.02 × 63.5 = 1.27 g
1.27
% purity = × 100 = 63.5%
2

9. (i) 2500 × 4.18 = 10450 kJ

10450  10 3
(ii) Mole of sucrose required = = 1.866
5.6  10 6

wt. of sucrose required  1.866 × 342 = 638.172 g


 1 mole of C12H22O11  12 mole of CO2
 1.866 moles of C12H22O11  1.866 × 12 moles of CO2
 22.392 moles of CO2
 1 mole of CO2  22.4 
 22.392 moles of CO2  22.4 × 22.392.
 501.58 
10. (a) Difference in flower colour is most likely due to environmental factors
(b) Perform cross breeding between the plants from Chandigarh and those from Shimla to find out
whether we get any pink flower or flowers with any shade of color between pink and white in the F1
generation
(c) Grow the plants from Chandigarh in Shimla and check whether they still produce white flowers of
bear pink flowers.

11. (a) In experiment A, ethanol fermentation occurs producing CO2, turning lime water milky. Since acid is
not produced the dye colour does not change.
In experiment B, lactic acid fermentation takes place, which produces acid but does not produce CO2.
Hence dye colour changes to yellow but the lime water does not turn milky.
In experiment C, since the lime water turns milky, ethanol fermentation is occurring. In addition, since
removal of air did not affect the reaction, the fermentation is anaerobic and yeast must be the organism
in the flask.
(b) In RBC’s lactic acid fermentation occurs.

12. (a) The result of the radio-carbon dating was correct.


Reason : Vehicles running on the highway beside the house emitted carbon dioxide from the combustion
of petrol or diesel, which are fossil fuels. The carbon in this carbon dioxide, coming from living material
that has been converted into petroleum millions of years ago, would get assimilated into the tissues of the
plant as it uses carbon dioxide from the surrounding atmosphere for photosynthesis. Therefore tissues of
the plant, when used for radio-carbon dating, would show the age of the plant to be many thousands of
years old.

(b) A simple experiment to test the validity of this explanation would be to collect seeds from the plant and
grow them in a plot of land away from the highway or other sources of carbon dioxide coming from the
burning or fossil fuels. Radio-carbon dating of plants growing from these seeds show them as young
plants.

PAGE # 8383

Вам также может понравиться